Sunteți pe pagina 1din 152

ADRIAN STAN

Dreptul de copyright:
Cartea downloadat de pe site-ul www.mateinfo.ro nu poate fi publicat pe un alt
site i nu poate fi folosit n
scopuri comerciale fr specificarea sursei i acordul autorului
Refereni tiinifici:

Prof. gr. I Mnzal Iorgu - inspector de matematic


Inspectoratul colar Judeean Buzu

Prof. gr. I Stanciu Neculai director Grupul colar Tehnic


Sf. Mc. SAVA, Berca

2
BREVIAR TEORETIC

DIVIZIBILITATE

Relaia de divizibilitate: a # b (sau ba)

c N respectiv Z astfel nct a=bc.


(a se divide cu b) sau (a este divizibil cu b) sau ( b divide pe a)
Proprieti:
1. aa , a Z *
2. 1a, a Z
3. ab, bc ac, a, b Z *
4. da,db da+b sau da-b
5. ab, ba a=b , a, b Z *
6. da dabc
7. ad, bd abd, dac a i b sunt prime ntre ele.
Descompunerea n factori primi:
n = p1 1 p 2 p3 ......... p k k N
a a2 a3 a

Numrul divizorilor lui n N este:


N= (a 1 + 1 )(a 2 + 1 )........ (a k + 1 ) .
Numrul divizorilor lui n Z este: N=2N
Numere prime
Numim numr prim orice numr natural mai mare dect 1,
care are numai divizori improprii-adic pe 1 i pe el nsui.

Criterii de divizibilitate:

3
Criteriul de divizibilitate cu 2
Un numr este divizibil cu 2 dac ultima sa cifr este par. Numerele
care sunt divizibile cu 2 se numesc numere pare.
Criteriul de divizibilitate cu 5
Un numr este divizibil cu 5 dac ultima sa cifr este 0 sau 5.
Criteriul de divizibilitate cu 4
Un numr este divizibil cu 4, dac numrul format de ultimele sale 2
cifre este divizibil cu 4.
Criteriul de divizibilitate cu 8
Un numr este divizibil cu 8, atunci cnd n umrul format de
ultimele sale 3 cifre este divizibil cu 8.
Criteriul de divizibilitate cu 25
Un numr este divizibil cu 25, dac numrul format de ultimele sale
2 cifre este divizibil cu 25, adic dac ultimele sale 2 cifre
sunt:00;25;50; 75.
Criteriul de divizibilitate cu 125
Un numr este divizibil cu 125, dac numrul format de ultimele sale
3 cifre este divizibil cu 125.
Criteriul de divizibilitate cu o putere a lui 10
Un numr este divizibil cu o putere a lui 10, dac ultimele sale n
cifre sunt zerouri.
Criteriul de divizibilitate cu 3
Un nr.este divizibil cu 3, dac suma cifrelor sale este un numr
divizibil cu 3.
Criteriul de divizibilitate cu 9
Un numr este divizibil cu 9, daca suma cifrelor sale este divizibil
cu 9.
Criteriul de divizibilitate cu 6
Un numr este divizibil cu 6, dac este divizibil cu 2 i cu 3.
Criteriul de divizibilitate cu 15
Un numr este divizibil cu 15, dac este divizibil cu 5 si cu 3.
Criteriul de divizibilitate cu 11
Un numr este divizibil cu 11, dac diferena dintre suma cifrelor
situate pe locurile impare si suma cifrelor situate pe locurile pare este
un numr divizibil cu 11.

Teorema mpririi cu rest n N.

4
Fie a, b N q, r N ,0 r b a. a = b q + r , b 0

Cel mai mare divizor comun al numerelor a i b (c.m.m.d.c) sau


(a,b) este cel mai mare numr la care se mpart exact si a si b i
este dat de produsul factorilor comuni, luai la puterea cea mai
mic.
1) (a;b)=d <=> a=dxa', b=dxb', (a';b')=1
2) (a;b)=d <=> d/a si d/b, oricare ar fi d' a.. d'/a si d'/b=> d'/d

Cel mai mic multiplu comun al numerelor a si b (c.m.m.m.c.) sau


[a,b] este cel mai mic numr care se mparte exact i la a i la b i
este dat de produsul factorilor comuni i necomuni luai la
puterea cea mai mare.
1)[a;b]=m <=> m=axm' , m=bxm'
2)[a;b]=m <=> a/m si b/m, oricare ar fi m', a.i. a/m' si b/m'=>m'/m

Relaia dintre c.m.m.m.c i c.m.m.d.c


[a,b](a,b)=ab
Dac p i q sunt prime atunci p n si q m sunt prime.

MULIMI. OPERAII CU MULIMI

N = {0 ,1, 2 ,3,......... .. n ,...... } N * = N \ {0}


Z = {...... n,.... 3,2,1,0,1,2,3,........., n,.......} Z * = Z \ {0}
a
Q = a, b Z , b 0 Q* = Q \ {0}
b

R \ Q = n n 0 nu este patrat perfect

R = Q ( R \ Q) = (,+) .

Fie A i B dou mulimi. Atunci:

5

A B = x x A sau x B . Reuniunea mulimilor.


A B = x x A si x B . Intersecia mulimilor.


A \ B = x x A si x B . Diferena mulimilor.


AxB = ( x, y ) x A si y B . Produsul cartezian.

Definiie: Se numete cardinal al unei mulimi finite , numrul de
elemente pe care-l are aceasta.
Principiul includerii i excluderii:
card(A B) =card (A) +card(B) card (AB).
Definiie: Se numete submulime al unei mulimi A, orice mulime
format cu elementele lui A.
Numrul tuturor submulimilor unei mulimi cu n elemente este 2n.

FRACII
a
Fracie: Se numete fracie, o expresie de forma , b 0 unde a se
b
numete numrtor iar b se numete numitor.
a
Fracie subunitar: Fracia , b 0 se numete fracie subunitar
b
a
dac a<b sau 1 .
b
a
Fracie echiunitar: Fracia , b 0 se numete fracie echiunitar
b
a
dac a=b sau =1.
b

6
a
Fracie supraunitar: Fracia , b 0 se numete fracie
b
a
supraunitar dac a>b sau 1 .
b
a c
Fracii echivalente: Dou fracii si se numesc echivalente i
b d
a c
scriem = dac a d = b c .
b d

A amplifica o fracie cu un numr natural, diferit de 0, nseamn a


nmuli att numrtorul ct i numitorul, cu acel numr.
a am
= , b 0, m 0.
b bm

A simplifica o fracie cu un numr natural, diferit de 0, nseamn a


mpri att numrtorul, ct i numitorul la acel numr.
a a:m
= , b 0, m 0.
b b:m
a
Fracie ireductibil: Fracia , b 0 se numete ireductibil,
b
dac nu se mai poate simplifica adic c.m.m.d.c(a,b)=1.

Compararea fraciilor:
a c a a a c
a c b c a d b c
b b b c b d

Numr raional :
a
Se numete numr raional , numrul reprezentat prin fracia i
b
toate fraciile echivalente cu aceasta.;

Operaii cu numere fracionare: Pentru a aduna sau scdea numere


fracionare reprezentate prin numitori diferii se aduc fraciile la

7
acelai numitor prin amplificarea fiecrei fracii cu ctul dintre
c.m.m.m.c al numitorilor i numitorul fraciei respective.
a c a + c a c ac a c a d
+ = ; = ; : = ;
b b b b d bd b d b c
0 m n m+ n
a a a a
= 1; = ;
b b b b
n
m
a a a
n mn
a m a
mn
a
1
b
: = ; = ; =
b b b b b b a
b ac + b
Introducerea unui ntreg n fracie: a = ;
c c
Fracii zecimale:
- Fracii zecimale: finite: {0,5;1,45;2,5;0,12345;......}
Transformarea fraciilor zecimale n fracii ordinare:
ab abc
a, b = ; a, bc = ;
10 100
- fracii zecimale -periodice: {1, (3);21,3(5);2,3( 4)......}
Transformarea fraciilor zecimale periodice n fracii ordinare:

ab a abc a
a, (b) = ; a, (bc) = ;
9 99
abc ab abcd ab
a, b(c ) = ; a, b(cd ) = ;
90 990
Scrierea n baza zece:
abcd = a 103 + b 102 + c 10 + d a-cifra miilor; b-cifra
sutelor; c-cifra zecilor; d-cifra unitilor;
a, efg = a 10 + e 101 + f 102 + g 103 =
= a 10 + e 0.1 + f 0.01+ g 0.001
a-cifra unitilor, e-cifra zecimilor; f-cifra sutimilor; g-cifra
miimilor.

8
Aflarea unei fracii dintr-un numr :
a a a c a c ac
din x = x ; din = = ;
b b b d b d bd

Procente: Un raport n care numitorul este 100, se numete raport


procentual si se noteaz de forma
p 1 p
= p 0 0 ; (sau p ) p 0 0 din x = x; p 0 0 din
100 100 100
a p a 1 a
= sau p
b 100 b 100 b

Operaii cu fracii zecimale:


La adunarea sau scderea fraciilor zecimale finite, numerele trebuie
aezate astfel nct virgula s fie sub virgul.
La nmulirea cu 10, 100, 1000 a unei fracii zecimale finite se
deplaseaz virgula spre dreapta cu 1,2,3 cifre.
La mprirea cu 10,100,1000, a unei fracii zecimale finite se
deplaseaz virgula spre stnga cu 1,2,3 cifre.
La nmulirea fraciilor zecimale finite, efectum nmulirea
obinuit dup care punem virgula de la dreapta spre stnga dup un
numr de zecimale egal cu numrul de cifre zecimale ale celor dou
numere;
La mprirea fraciilor zecimale finite se vor nmulii ambele numere
cu puteri ale lui 10 astfel nct s mprim numere fr virgul.

9
Ultima cifr a unui numr n
U (abc ) = U (c n )

U(cn) n=4k+1 n=4k+2 n=4k+3 n=4k

1n 1 1 1 1
2n 2 4 8 6
3n 3 9 7 1
4n 4 6 4 6
5n 5 5 5 5
6n 6 6 6 6
7n 7 9 3 1
8n 8 4 2 6
9n 9 1 9 1

Proporii: Egalitatea a dou rapoarte se numete proporie:


a c
Proprietatea fundamental a proporiilor: = a d = bc
b d
Proporii derivate:

a c a b a b ab cd
= = = , =
b d c d ba d b b d
2 2
a c a c
2
= 2 = a a+c
=
b d b:m d :m b b+c
am cm a c
= = a a+c
=
b d b:m d :m b b+c

am cm a c
= a
=
ca =
b d b d b b:m d :m

10
Sir de rapoarte egale:
Mrimile (a1,a2,a3,......, an ) i ( b1,b2,....,bn ) sunt direct proporionale
a1 a 2 a
= = ....... n
b1 b2 bn
Mrimile (a1,a2,a3,......, an ) i (b1,b2,....,bn ) sunt invers
proporionale a1 b1 = a 2 b2 = .....a n bn

Probabiliti

Probabilitatea realizrii unui eveniment este dat de raportul dintre


numrul cazurilor favorabile realizrii evenimentului i numrul
cazurilor egal posibile.

Modulul numerelor reale Proprieti:


a, a0

a def 0, a = 0

a, a 0

1. a 0, a R ; 2. a = 0, a = 0;

3. a = a , a R ; 4. a = b , a = b ;
a a
5. a b = a b ; 6. = ; b0
b b
n
= , x R , n N
n
7. x x
a b a + b , a, b R
8.
9. a b a b a + b ; a, b R

11
10. x = a, x = a, a 0 ;

11. x a, x [ a, a ], a 0 ;
a+b+ ab
12. max (a, b) = , a, b R ;
2
a+b ab
min (a, b) =
2
13. x a, x [, a ] [a,+], a 0 ;
14. a1 a 2 ... a n a1 + ... + a n , in R .

Puteri cu exponent ntreg


Fie a R, n Z

a n def a a ......
a a

n factori

Fie a, b R, a, b 0, m, n Z *

1. a o = 1; a 1 = a;0 n = 0; 5 . ( a m ) n = a m n
1
2. a m + n = a m a n 6. a n =
an
n
a an
3. ( a b ) = a b
n n n
7. = n
b b
am
4. n = a m n ; 8. a m = a n m = n.
a

n n
1, n par a b
9. (-1) =
n
10. =
1, n impar b a

12
Proprietile radicalilor

Fie a , b R +

1. a 2 = a 0,
2. a b = a b
a a
3. = , b0
b b
n

4. a = ( a) = a
n n 2

a + a2 b a a2 b
5. a b =
2 2
unde a-b=k . (formula radicalilor dubli)
6. Dac a N a = k2
7. a b = a 2b
8. a 2b = a b
9. x a y b = xy ab
10. x a y a = ( x y ) a
x a x a
11. = , y, b 0
y b y b
( )
12. x a
n
= xn an
m
13. Fie a R \ Q . Atunci a= Q m = n = 0
n
14. Dac m, n Q i a nu e ptrat perfect i
m+n a =0m= n =0
15. Dac a, b N i a + b Q a N , b N
16. Dac a i b nu sunt ptrate perfecte a + b Q

13

17. Dac a, b Q+ i , Q a..

a + b Q a Q , b Q + +

18. Dac a, b Q a.. b R \ Q a b R \ Q i
+
a bR\Q

19. Dac a Q i b R \ Q a + b R \ Q i
a bR \Q
Raionalizri
x x a x x b
= , =
a a a b ab
x x(a b ) x x( a + b )
= , =
a+ b a2 b a b a2 b

Medii

x+ y
Media aritmetic ma =
2
Media geometric m g = x y , x, y R * +
px+q y
Media ponderat m p = ; p, q N * ponderile
p+q
2 2 xy
, x, y R
*
Media armonic m h = =
1 1 x+ y
+
x y

14
ECUAII

b
a x + b = 0 a x = b x = , a 0.
a
x = a x = a , a 0. ;
2

b b 2 4ac
a x 2 + b x + c = 0 x1, 2 =
2a
a 0. b 2 4ac 0

x = a, a 0 x = a.
[x] = a a x a + 1 x [a, a + 1) .

16. PROCENTE

p
p % din N = N
100
p
Raportul se numete raport procentual iar p se numete
100
procent.
Aflarea unui numr cnd cunoatem p% din el.
p 100
din x =a x = a .
100 p
Aflarea raportului procentual:
Ct la sut reprezint numrul a din N ?
a 100
p % din N =a p = .
N

S pn
D= . Dobnda obinut prin depunerea la banc a unei
100 12
sume S de bani pe o perioad de n luni cu procentul p al dobndei
anuale acordate de banc .

15
CALCUL ALGEBRIC

Reguli de calcul n R

1. (a + b ) = a 2 + 2ab + b 2 ;
2

2. (a b ) = a 2 2ab + b 2 ;
2

3. (a+b)(a-b= a 2 -b 2 ;
4. (a + b + c ) = a 2 + b 2 + c 2 + 2ab + 2bc + 2ca
2

5*. (a + b ) = a 3 + 3a 2 b + 3ab 2 + b 3 ;
3

6*. (a b ) = a 3 3a 2 b + 3ab 2 b 3 ;
3

7*. a 3 b 3 = (a b)(a 2 + ab + b 2 ) ;
8*. a 3 + b 3 = (a + b)(a 2 ab + b 2 ) .

Descompuneri n factori

1. Metoda factorului comun


ab+ac-ad=a(b+c-d)
ax+ay+by+by=a(x+y)+b(x+y)=(x+y)(a+b)

2. Utilizarea formulelor de calcul prescurtat


a2+2ab+b2 =(a+b)2
a2-2ab+b2 =(a-b)2
a2+b2+c2+2ab+2bc+2ac=(a+b+c)2
a2-b2=(a-b)(a+b)

Rapoarte de numere reale reprezentate prin litere

a ac
Amplificarea = , b, c 0
b bc
a a:c
Simplificarea = , b, c 0
b b:c

16
a c ac
Adunarea sau scderea = , b0
b b b
a c ac
nmulirea = , b, d 0
b d bd
a c a d
mprirea : = b, c, d 0
b d b c
a an
Puterea cu exponent natural ( ) n = n , b 0, n N *
b b
Puterea cu exponent ntreg negativ
n n
a b
= , a, b 0, n N * .
b a

17
FUNCII

Sistem de axe ortogonale

Definiie: Se numete funcie, un triplet de forma (A,B,f), unde A se


numete domeniu de definiie (mulimea de unde funcia ia valori), B
se numete codomeniu (mulimea valorilor funciei), iar f se numete
lege de coresponden(face ca fiecrui element din A s-i corespund
un unic element din B).
Notaie: f:AB
{
Imaginea funciei este mulimea Im f= y B y = f ( x), x A }

18
Exemplu: f(x)=x2

Graficul unei funcii f:AB este mulimea


{ }
G f = ( x, f ( x ) ) x A A B .
Condiia ca un punct M(a,b) s aparin graficului lui f.
M ( a, b) G f f ( a ) = b .
Reprezentarea grafic:
f: RR , f(x)=ax+b graficul este o dreapt;

f (x) not y y=ax+b

b b
OX : y = 0 x = A( ,0)
a a
OY : x = 0 y = b B(0, b)

19
f: IR, unde I este un interval; dac I este mrginit, atunci graficul
este un segment iar dac I este nemrginit la un capt i mrginit la
cellalt atunci graficul este o semidreapt.

f:AR, unde A este o mulime finit de puncte atunci graficul lui f


este tot o mulime finit de puncte.

Condiia ca trei puncte A(x1,y1), B(x2,y2), C(x3,y3) s fie coliniare:


- se determin funcia f(x)=ax+b, al crui grafic este
determinat de dou puncte i se verific dac i cel de-al
treilea punct aparine graficului lui f;
- cu ajutorul lungimilor distanelor dintre puncte , se verific
dac lungimea segmentului cel mai mare este egal cu suma
lungimilor celorlalte dou segmente.

UNITI DE MSUR

Multiplii i submultiplii metrului- uniti de msur pentru


lungime

km hm dam m dm cm mm

Multiplii metrului Submultiplii metrului

Multiplii i submultiplii m2- uniti de lungime pentru arie

km2 hm2 dam2 m2 dm2 cm2 mm2

Multiplii metrului ptrat Submultiplii metrului 2

1 ha =1 hm2=10000m2 1 ar = 100m2

20
Multiplii i submultiplii m3-uniti de msur pentru volum

km3 hm3 dam3 m3 dm3 cm3 mm3

Multiplii metrului cub Submultiplii metrului3

Multiplii i submultiplii litrului- uniti de msur pentru


capacitate

kl hl dal l dl cl ml

Multiplii litrului Submultiplii litrului

1 dm3=1 l

Multiplii i submultiplii gramului- uniti de msur pentru


mas
kg hg dag g dg cg mg

Multiplii gramului Submultiplii gramului

1 q=100kg 1 t = 1000kg

Uniti de msur pentru timp

1 min=60 s
1 h =60 min=3600s
1 zi =24 h
1 an =365 zile sau 366 zile(an bisect)
1 deceniu = 10 ani; 1 secol =100 ani; 1 mileniu = 1000 ani.

21
UNGHIURI

Def 1. Dou unghiuri proprii se numesc opuse la vrf dac laturile


lor formeaz dou perechi de semidrepte opuse.

Def 2. Dou unghiuri se numesc complementare dac suma


msurilor lor este de 90 i se numesc suplementare dac suma
msurilor lor este de 180.

Def 3. Dou drepte din acelai plan care nu au nici un punct comun
se numesc drepte paralele.

Postulatul lui Euclid: Printr-un punct exterior unei drepte trece o


singur dreapt paralel la dreapta dat.

Def 4. Dou drepte concurente sunt perpendiculare dac unul din


unghiurile care se formeaz n jurul punctului lor comun este de 90.

Def 5. Dou drepte paralele intersectate de o secant formeaz :


- unghiuri alterne interne congruente: 4 cu 6, 3 cu 5;
- unghiuri alterne externe congruente: 1 cu 7, 2 cu 8;
- unghiuri corespondente congruente: 4 cu 8, 3 cu 7, 2 cu 6, 1
cu 5;
- unghiuri interne sau externe de aceeai parte a secantei
suplementare: 1 cu 4, 1 cu 8, 4 cu 5, 5 cu 8.

22
Def 6. Se numete unghi ascuit, unghiul a crui msur este mai
mic de 90 .

Def 7. Se numete unghi obtuz, unghiul a crui msur este mai


mare de 90 .

Def 8. Se numete unghi nul, unghiul a crui msur este de 0 .

Def 9. Se numete unghi alungit , unghiul cu semidreptele n


prelungire i cu msura de 180.

Teorema 1. Suma msurilor unghiurilor n jurul unui punct din plan


este de 360.

Teorema 2. Suma msurilor unghiurilor n jurul unui punct pe o


dreapt este de 180.

Def 10. Dou unghiuri se numesc congruente dac au aceeai


msur.

Def 11. Dou unghiuri se numesc adiacente dac au vrful comun i


o latur comun situat n interiorul unghiului format de celelalte
dou laturi ale unghiurilor.

23
Triunghiul. Linii importante n triunghi.

Aria triunghiului:

BC AD
AABC =
2
AABC = p ( p a )( p b)( p c)
AB + AC + BC
unde p = (semiperimetru)
2
AB AC sin A
AABC =
2

nlimea este segmentul de dreapt ce unete vrful triunghiului


cu piciorul perpendicularei dus din vrf pe latura opus; Intersecia
nlimilor este un punct ce se numete ortocentru.
Mediana este segmentul de dreapt ce unete vrful triunghiului
cu mijlocul laturii opuse; intersecia medianelor este un punct ce se
numete centru de greutate care se afl la o treime fa de baz i
dou treimi fa de vrf.

24
Lungimea medianei n funcie de laturi:
2( AB 2 + AC 2 ) BC 2
AM 2 = .
4
Mediana mparte un triunghi n dou triunghiuri echivalente, care au
aceeai arie:
AABM=AAMC
Bisectoarea este segmentul de dreapt ce mparte unghiul
triunghiului n dou unghiuri congruente. Intersecia nlimilor se
noteaz cu I i se numete centrul cercului nscris n triunghi.
Orice punct aflat pe bisectoarea unghiului se afl la egal distan de
laturile unghiului.
Mediatoarea laturii unui triunghi este dreapta perpendicular pe
latura triunghiului exact prin mijlocul ei. Intersecia mediatoarelor se
noteaz cu O i se numete centul cercului circumscris triunghiului.
Orice punct aflat pe mediatoarea laturii unui triunghi se afl la egal
distan de capetele segmentului.
Linia mijlocie n triunghi este segmentul de dreapt ce unete
mijloacele a dou laturi ale triunghiului; Linia mijlocie este paralel
cu baza i jumtate din ea.

Teorema liniei mijlocii:


1
MN-linie mijlocie MN BC , MN = BC
2

25
Teorema reciproc asupra liniei mijlocii:
1
Dac AM MB i MN BC AN NC , MN = BC
2
Dac MN este linie mijlocie n triunghi atunci mijloacele nlimii,
bisectoarei i medianei aparin liniei mijlocii;

Metoda triunghiurilor congruente:


Pentru a arta c dou segmente sau dou unghiuri sunt congruente
trebuie artat c triunghiurile din care fac parte sunt congruente.

Cazurile de congruen ale triunghiurilor oarecare:


I. L.U.L.
II. U.L.U
III. L.L.L.
IV. L.U.U
Cazurile de congruen ale triunghiurilor dreptunghice:
I. catet-catet
II. catet-ipotenuz
III. ipotenuz-unghi ascuit
IV. catet-unghi ascuit

26
ASEMNAREA TRIUNGHIURILIR

Definiie: Trei sau mai multe drepte paralele se numesc echidistante


dac sunt situate la distane egale.

Teorema paralelelor echidistante:


Dac mai multe drepte paralele determin pe o secant segmente
congruente, atunci ele determin pe orice alt secant segmente
congruente.
d1|| d2 || d3 ||.......|| dn i A1A2 A2A3 A3A4 ..............An-1An
B1B2 B2B3 B3B4 ................Bn-1Bn
A3 An
A1 A2

B1 B2 B3 Bn
Definiie: Prin raportul a dou segmente, se nelege raportul
lungimilor lor, exprimate cu aceeai unitate de msur.
Definiie: Segmentele AB, BC, AC sunt proporionale cu segmentele
AB, BC, AC dac ntre lungimile lor exist o relaie de
AB BC AC
proporionalitate de forma: = = .
A' B' B' C ' A' C '
Teorema lui Thales:
O paralel dus la una din laturile unui triunghi determin pe
celelalte dou laturi sau pe prelungirile lor segmente proporionale.
AM AN AM AN MB NC
MN BC = sau = sau = (*)
AB AC MB NC AB AC

27
Teorema paralelelor neechidistante:
Mai multe drepte paralele determin pe dou secante segmente
proporionale.
BD AB
Teorema bisectoarei: BA D DA C = .
DC AC
ac a b
Dac BC= a, AC=b, AB=c, atunci BD = , CD = ,
b+c b+c
bc
AD = [(b + c) 2 a 2 ] - lungimea bisectoarei
(b + c) 2

Teorema reciproc a lui Thales:


Dac o dreapt determin pe laturile unui triunghi segmente
proporionale, atunci ea este paralel cu cea de a treia latur a
triunghiului.
AM AN AM AN MB NC
Dac = sau = sau = atunci
AB AC MB NC AB AC
MN BC .
Definiie:
Dou triunghiuri se numesc asemenea dac au toate laturile
proporionale i unghiurile congruente .

28
M
A

C N P
B

A M , B N , C P

ABC ~ MNP AB AC BC
= =
MN MP NP

Teorema fundamental a asemnrii:


O paralel la una din laturile unui triunghi formeaz cu celelalte
laturi sau cu prelungirile lor un triunghi asemenea cu cel dat.
MN BC AMN ~ ABC .

Criteriile de asemnare a triunghiurilor.


A M si B N ABC ~ MNP (u.u)
AB AC
= si A M ABC ~ MNP (l.u.l)
MN MP
AB AC BC
= = ABC ~ MNP (l.l.l)
MN MP NP

Proprieti ale asemnrii triunghiurilor:


1. ABC ~ ABC .(Reflexivitatea relaiei de asemnare)
2. ABC ~ AMN AMN ~ ABC .(Simetria)
3. ABC ~ AMN , AMN ~ PQR ABC ~
PQR .(Tranzitivitatea)

29
4. Dou triunghiuri isoscele sunt asemenea au o pereche de
unghiuri congruente;
5. Dou triunghiuri echilaterale sunt asemenea ntodeauna;
6. Dou triunghiuri dreptunghice sunt asemenea au o pereche de
unghiuri ascuite congruente;
7. Dou triunghiuri cu laturile respectiv paralele sunt asemenea;
8. Dou triunghiuri cu laturile respectiv perpendiculare sunt
asmenea;
9. Dac dou triunghiuri sunt asemenea atunci raportul de asemnare
al laturilor este egal cu:
- raportul medianelor;
- raportul bisectoarelor;
- raportul nlimilor;
- raportul razelor cercurilor nscrise;
- raportul razelor cercurilor circumscrise;
10. Raportul ariilor a dou triunghiuri asemenea este egal cu ptratul
raportului de asemnare al laturilor.

30
RELAII METRICE
N TRIUNGIUL DREPTUNGHIC

Formula distanei dintre punctele A(x1,y1), B(x2,y2):


AB= ( x 2 x1 ) + ( y 2 y1 )
2 2

Teorema lui Pitagora.


ntr-un triunghi dreptunghic, suma ptratelor catetelor este egal cu
ptratul ipotenuzei.
BC2=AB2+AC2;

Teorema reciproc a lui Pitagora:


Dac ntr-un triunghi, suma ptratelor a dou laturi este egal cu
ptratul celei de-a treia laturi, atunci triunghiul este dreptunghic.

Dac BC2=AB2+AC2 m( A ) = 90 0

31
Teorema nlimii:
nlimea corespunztoare ipotenuzei este media geometric a
proieciilor catetelor pe ipotenuz.
AD 2 = BD DC
AB AC
Teorema a II-a a nlimii: AD = .
BC
Teorema catetei:
Cateta este media geometric dintre ipotenuz i proiecia catetei pe
ipotenuz.
AB2 = BC BD ; AC2 = BC CD
Teorema unghiului de 300:
Cateta opus unghiului de 300 este jumtate din ipotenuz. Dac
BC
m(C ) = 30 0 AB = ;
2
Teorema unghiului de 150:
nlimea corespunzroare ipotenuzei ntr-un triunghi dreptungic cu
un unghi de 150 este un sfert din ipotenuz.
BC
Dac m(C ) = 150 AD =
4
Teorema medianei ntr-un triunghi dreptunghic:
Mediana ntr-un triunghi dreptunghic este jumtate din ipotenuz;
BC
AM median AM = .
2
n orice triunghi dreptunghic are loc relaia:
sin2 +cos2=1

Aria triunghiului dreptunghic:

BC AD AB AC
AABC= sau
2 2

32
Rapoarte constante n triunghiul dreptunghic:

300 450 600


AB 1 2 3
sin C =
BC 2 2 2
3 2 1
AC
cos C = 2 2 2
BC
3
AB 1 3
tgC = 3
AC
3
AC 3 1
ctgC = 3
AB

Teorema cosinusului: a2=b2+c2-2bccosA

33
Teorema lui Pitagora generalizat:
AB2=BC2+AC2-2BC.DC m(C )90 0
AB2=BC2+AC2+2BC.DC m(C )90 0
AC2=AB2+BC2-2BC.BD m( B )90 0
AC2=AB2+BC2+2BC.BD m( B )90 0

Distana de la un punct M(x0,y0) la o dreapt de ecuaie:


ax+by+c=0;
a x0 + b y 0 + c
d (M , d ) =
a2 + b2

34
PATRULATERE

DEFINIIE. Se numete patrulater convex patrulaterul n care


segmentul determinat de oricare dou puncte ale lui este interior
patulaterului.

PARALELOGRAMUL
Definiie: Se numete paralelogram, patulaterul convex care are
laturile opuse paralele.
Teoreme:
1. Un patrulater este paralelogram dac i numai dac laturile opuse
sunt congruente dou cte dou.
2. Un patrulater este paralelogram dac i numai oricare dou
unghiuri opuse sunt congruente i oricare dou unghiuri consecutive
sunt suplementare.
3. Un patrulater este paralelogram dac i numai dac diagonalele
se njumtesc.
4. Un patrulater este paralelogram dac i numai dac dou laturi
opuse sunt paralele i congruente.
5. Un patrulater este paralelogram dac i numai dac oricare dou
unghiuri consecutive sunt suplementare.

35
Aria paralelogramului:

AABCD= BC AE

AC BD sin
AABCD=
2

AABCD= AB BC sin( AB C )

DREPTUNGHIUL
Definiie: Se numete dreptunghi, paralelogramul cu un unghi drept.
Proprieti:
ntr-un dreptunghi, toate unghiurile sunt de 900.
ntr-un dreptunghi, diagonalele sunt congruente.
Teorem: Dac un paralelogram are diagonalele congruente, atunci
el este dreptunghi.

Aria dreptunghiului:

AABCD= AB BC

36
ROMBUL

Definiie: Se numete romb, paralelogramul care are dou laturi


consecutive congruente.
Teorem:
1. Un patrulater este romb dac i numai dac are toate laturile
congruente.
2. Un paralelogram se numete romb dac i numai dac are
diagonalele perpendiculare.
3. Un paralelogram se numete romb dac i numai dac o diagonal
este bisectoarea unui unghi.

Aria rombului:

Dd
AABCD= , unde D este diagonala mare iar d este diagonala
2
mic.

37
PTRATUL

Definiie: Se numete ptrat, paralelogramul care este i romb i


dreptunghi.
Proprieti:
1. Toate unghiurile ptratului sunt drepte.
2. Diagonalele sunt bisectoarele unghiurilor ptatului.
3. Diagonalele sunt perpendiculare i congruente.

Aria ptratului:
AABCD=l2

TRAPEZUL
Definiie: Se numete trapez, patrulaterul care are dou laturi opuse
paralele, iar celelalte laturi neparalele.

Proprieti.
1. Patrulaterul ABCD este trapez isoscel dac i numai dac

A D si B C .
2. Patrulaterul ABCD este trapez isoscel dac i numai dac
diagonalele sunt congruente.

38
Aria trapezului:
( B + b) h
AABCD= , unde B este baza mare, b este baza mic, iar h
2
este nlimea trapezului.

B+b
Linia mijlocie: MN= ;
2
Bb
PQ=
2

39
CERCUL

1. Elemente n cerc

Definiii:
Se numete cerc de centru O i raz r i scriem C(O,r)
mulimea tuturor punctelor din plan situate la distana r fa de
punctul O.
C (O , r ) = { M OM = r,r R+ }
Segmentul care unete dou puncte de pe cerc se numete
coard; Coarda care trece prin centrul cercului se numete
diametru, iar capetele diametrului se numesc puncte diametral
opuse.
Poriunea dintr-un cerc determinat de dou puncte distincte ale
cercului se numete arc de cerc. Dac extremitile unui arc sunt
diametral opuse, atunci arcul se numete semicerc.
Se numete interiorul cercului, mulimea punctelor aflate fa
de centru la distane mai mici dect raza cercului, iar exteriorul
cercului reprezint mulimea punctelor situate fa de centru la
distane mai mari dect raza cercului.
Mulimea punctelor cercului C(O,r) reunit cu interiorul cercului
se numete disc de centru O i raz r: D(o,r).
Se numete unghi la centru, unghiul cu vrful n centrul unui
cerc.

Msura n grade a unui arc este egal cu msura unghiului la


centru corespunztor.

40

m( ADB) = m( AO B)
Msura n grade a unui semicerc este de 1800, iar a unui cerc este de
3600.

1800.................. rad
x0
x0.....................y rad y=
1800
Se numete sector de cerc, o poriune dintr-un cerc cuprins ntre
dou raze ale sale i arcul pe care l subntind.
Se numesc arce congruente , arcele care au aceeai msur.
Teoreme:
1. ntr-un cerc, arcelor congruente le corespund coarde congruente i
reciproc.
 
AB CD [ AB] [CD ]

2. ntr-un cerc, diametrul perpendicular pe o coard trece prin


mijlocul arcului subntins de coard.
3. Dou coarde ale unui cerc sunt congruente dac i numai dac sunt
egal deprtate de centru. [AB ] [CD ] d (O, AB ) = d (O, CD ) .
4. Dac dou coarde ale unui cerc sunt paralele, atunci arcele
cuprinse ntre ele sunt congruente.

41
2. Poziiile relative ale unei drepte fa de un cerc.

a) h dreapt secant d (O, h) r


b) h dreapt tangent d (O, h) = r
c) h dreapt exterioar d (O, h) r
Tangente dintr-un punct exterior la un cerc

1. Dintr-un punct exterior unui


cerc se pot duce dou tangente
i numai dou la cerc;
2. PT PT ' ;
3. PO este bisectoarea
unghiului TPT;
4. PO este mediatoarea
segmentului TT
5. Msura unui unghi cu vrful
pe cerc, care are una din laturi
secant i cealalt tangent la cerc, este jumtate din msura arcului
cuprins ntre laturile sale.

42
Unghiul nscris n cerc
Se numete unghi nscris n cerc, unghiul cu vrful pe cerc ale crui
laturi includ dou coarde ale cercului.


m( BC )
m( BAC ) = ;
2

m( BO C ) = m( BC )

 
m( BD) m( AC )
m( BPD) =
2

 
m ( A C ) + m ( B D)
m( AP C ) =
2

43
3. Poziiile relative ale dou cercuri

a) cercuri secante r-r<OO<r+r, r>r


b) cercuri tangente interioare OO=r-r, r>r
c) cercuri exterioare OO>r+r
d) cercuri tangente exterioare OO=r+r
e) cercuri concentrice au acelai centru
f) cercuri interioare OO<r-r, r>r

4.Triunghi nscris n cerc. Patrulater nscris n cerc


Definiii:
Cercul care conine cele trei vrfuri ale unui triunghi se numete
cercul circumscris triunghiului.
Centrul cercului circumscris unui triunghi este punctul de intersecie
a mediatoarelor triunghiului.

44
Patrulaterul cu vrfurile pe cerc se numete patrulater nscris n
cerc(patrulater inscriptibil).
Teorem:
1. ntr-un patrulater nscris n cerc, diagonalele formeaz cu laturile
opuse perechi de unghiuri congruente.
2. Unghiurile opuse ale unui patrulater nscris n cerc sunt
suplementare.

Raza cercului circumscris unui triunghi n funcie de aria triunghiului


i de laturile sale este:
abc
R=
4S

5. Poligoane regulate nscrise n cerc.

Definiie: Un poligon se numete regulat dac este convex, are toate


laturile congruente i toate unghiurile congruente.

Distana de la centrul poligonului regulat la oricare dintre laturile


sale se numete apotema poligonului (a).

45
Perimetrul poligonului regulat cu n laturi este P = n l unde l este
latura poligonului;
a P
Aria poligonului regulat cu n laturi este A= , unde a este
2
apotema ;
(n 2) 1800
Msura unui unghi este un =
2
Triunghiul echilateral

R
l3 = R 3 a3 = sau
2
1
a3 = h
3
l2 3
Aria triunghiului. A3=
4

Ptratul

R 2 L
l 4= R 2 ; a4 = =
2 2

46
Hexagonul regulat

l 6 = R;
R 3
a6 =
2
3
A3 = 6l 2
4

Un patrulater ABCD este inscriptibil dac unghiul dintre o


diagonal a sa i o latur este egal cu unghiul dintre cealalt
diagonal i latura opus, sau dac dou unghiuri opuse fac 1800.

Un triunghi este circumscris unui cerc sau un cerc este nscris n


triunghi dac distanele de la centrul cercului la toate cele trei laturi
sunt egale.

Raza cercului nscris ntr-un triunghi n funcie de aria triunghiului i


de semiperimetrul triunghiului.

S a+b+c
r= unde p = , a,b,c sunt laturile triunghiului.
p 2

47
6. Lungimea cercului. Aria cercului

Lungimea cercului: lc = 2 R
A= R
2
Aria cercului:

uR
Lungimea arcului de cerc AMB: l = = uR
180 0
AMB

uR 2 uR 2
Aria sector de cerc AOB: AAOB = =
3600 2

48
Elemente de geometrie n spaiu

Cap I. PUNCTE, DREPTE, PLANE

1. RELAII NTRE PUNCTE, DREPTE I PLANE

Noiunile fundamentale ale geometriei sunt:punctul,dreapta,


planul, distana i msura unghiurilor.
Definiie: Se numete axiom un adevr simplu matematic care nu
se demonstreaz deoarece se verific n natur.
Axiome: 1.Spaiul este o mulime infinit de puncte.
2. Dreptele i planele sunt submulimi ale spaiului.
3. Orice plan conine cel puin trei puncte necoliniare (nu
sunt situate pe aceeai dreapt)
4. Exist patru puncte care nu aparin aceluiai plan
(necoplanare).
5. Prin orice dou puncte distincte trece o singur dreapt.
6. Prin orice trei puncte necoliniare trece un singur plan.
Definiie: O dreapt este inclus ntr-un plan dac orice punct al
dreptei aparine planului.
Teorema 1.1 Dac dou puncte distincte ale unei drepte aparin unui
plan atunci dreapta este inclus n acel plan.
Teorema 1.2 Dac dou plane au un punct comun atunci ele au o
dreapt comun.
2. DETERMINAREA PLANULUI

Conform axiomei 6 trei puncte necoliniare determin un plan , n


plus urmtoarele teoreme ne indic alte situaii de determinare a
planului.
Teorema 2.1 O dreapt i un punct exterior ei determin un plan.
Teorema 2.2 Dou drepte paralele determin un plan.
Teorema 2.3 Dou drepte concurente determin un plan.

49
A*

fig 1 fig 2 fig 3

3. CORPURI GEOMETRICE

Definiie: Corpurile geometrice se definesc ca fiind mulimea


tuturor punctelor, dreptelor i planelor din spaiul cu trei dimensiuni
care se gsete n interiorul unei suprafee nchise, inclusiv punctele ,
dreptele i poriunile de plan care se gsesc pe aceast suprafa.
Mulimea tuturor punctelor din spaiu care se gsesc n interiorul
suprafeei corpului se numete volumul corpului.

POLIEDRE

Definiie: Corpurile mrginite numai de suprafee plane se numesc


poliedre. Poligoanele plane care mrginesc poliedrul se numesc
fee(laterale), segmentele comune feelor se numesc muchii i
capetele acestor segmente , vrfuri.

4. POZIIILE RELATIVE A DOU DREPTE N SPAIU

Definiie: Se numesc drepte coplanare , dreptele care sunt situate n


acelai plan. n caz contrar se numesc drepte necoplanare.
Dou drepte coplanare pot fi:
1. paralele (nu au nici un punct n comun).
2. concurente au un singur punct n comun).
3. identice (mulimea punctelor lor coincid)

50
Axioma paralelelor: Printr-un punct exterior unei drepte se poate duce cel
mult o paralel la dreapta dat.

Teorema 4.1: Dou drepte distincte din spaiu, paralele cu o a treia sunt
paralele ntre ele.

5. POZIIILE RELATIVE ALE UNEI DREPTE FA DE UN


PLAN

Definiie: O dreapt este paralel cu un plan dac dreapta i planul


nu au puncte comune.
Definiie: O dreapt este secant unui plan dac dreapta are un
singur punct comun cu planul.
Definiie: O dreapt este inclus n plan dac orice punct al ei
aparine planului.

Teorema 5.1: O dreapt paralel cu o dreapt din plan este paralel


cu planul sau coninut n el.

51
6. POZIIILE RELATIVE A DOU PLANE

Planele pot fi de trei feluri: paralele (adic nu au nici un punct


comun) , secante( adic au n comun o dreapt dup care se
intersecteaz) sau identice( mulimea punctelor lor coincid).

OBSERVATII:
1. O dreapt paralel cu un plan nu este neaprat paralel cu orice
dreapt din plan.
2. Dou drepte paralele cu un plan nu sunt neaprat paralele ntre
ele.
3. Dou drepte situate n plane paralele nu sunt neaprat paralele.
4. Dou plane, paralele cu o dreapt, nu sunt neaprat paralele ntre
ele.

7. TEOREME DE PARALELISM

Teorema 7.1 Dac o dreapt d este paralel cu un plan i coninut ntr-


un plan care se intersecteaz dup o dreapt g,
atunci d i g sunt paralele.
Teorema 7.2 Dndu-se dou plane paralele, orice dreapt dintr-un
plan este paralel cu al doilea plan.
Teorema 7.3 Dac dou plane paralele sunt tiate de un al treilea
plan atunci dreptele de intersecie sunt paralele.
Urmtoarea teorem stabilete cnd dou plane sunt paralele.

52
Teorema 7.4 Dac un plan conine dou drepte concurente paralele
cu un alt plan atunci cele dou plane sunt paralele.
Teorema 7.5 ( Tranzitivitatea relaiei de paralelism mtre plane).
Dou plane distincte paralele cu un al treilea plan sunt paralele ntre
ele.
Teorema 7.6 Dou plane paralele determin pe dou segmente
paralele pe care le intersecteaz segmente congruente.
Teorema 7.7 (Teorema lui Thales n spaiu)
Mai multe plane paralele determin pe dou drepte oarecare pa care
le intersecteaz segmente respectiv proporionale.

8. UNGHIUL A DOU DREPTE N SPAIU

Definiie: Dou unghiuri se numesc suplementare dac suma


msurilor lor este de 180 i se numesc complementare dac suma
msurilor lor este de 90.
Teorema 8.1 Dou unghiuri din acelai plan sau din plane diferite cu
laturile respectiv paralele sunt congruente sau suplementare.

Unghiuri congruente
respectiv Unghiuri congruente respectiv
suplementare n suplementare n plane diferite
acelai plan

53
Definiie: Prin unghiul a dou drepte n spaiu se nelege unghiul
de msur mai mic sau cel mult egal cu 90 cu vrful n orice
punct al spaiului format prin ducerea de paralele la dreptele date
prin acel punct.

Observaii:
1. De obicei, vrful unghiului a dou drepte din spaiu se ia pe una
din drepte.
2. Dac dreptele sunt paralele, atunci ele formeaz un unghi de 0.
3. Dac dreptele sunt concurente atunci ele formeaz un unghi plan

9. DREAPTA PERPENDICULAR PE UN PLAN

Definiie: Dou drepte din spaiu( concurente sau necoplanare) care


formeaz ntre ele un unghi drept se numesc drepte perpendiculare.

54
Definiie: Se numete dreapt perpendicular pe un plan , dreapta
care este perpendicular pe orice dreapt din plan.

Teorema 9.1 (Criteriul de perpendicularitate.) Dac o dreapt este


perpendicular pe dou drepte concurente dintr-un plan atunci ea este
perpendicular pe plan.
Teorema 9.2 Dintr-un punct exterior se poate duce pe un plan o
perpendicular i numai una.
Teorema 9.3 Dou drepte distincte perpendiculare pe un acelai plan
sunt paralele ntre ele.
Teorema 9.4 Printr-un punct se poate duce un plan i numai unul
perpendicular pe o dreapt dat.
Definiie: Prin distana dintre dou puncte din spaiu se nelege
lungimea segmentului determinat de cele dou puncte.
Lungimea diagonalei cubului: D= a 3
Lungimea diagonalei paralelipipedului dreptunghic
2 2 2
D= a +b +c

Definiie: Distana dintre un punct i un plan este lungimea


segmentului determinat de punct i de plan pe perpendiculara dus
din acel punct pe plan.

55
Definiie: Distana dintre dou plane paralele este lungimea segmentului
determinat de cele dou plane pe o perpendicular comun.

10. SECIUNI PARALELE CU BAZA N POLIEDRE

Definiie: A seciona o prism cu un plan paralel cu baza nseamn a


intersecta feele laterale cu un plan paralel cu baza. Seciunea
rezultat este un poligon asemenea cu cel de la baz chiar mai mult
este congruent cu acesta. n urma secionrii se formeaz dou
prisme de acelai tip cu prisma iniial.

Definiie: Se numete trunchi de piramid corpul geometric obinut


prin secionarea unei piramide cu un plan paralel cu baza, situat ntre
baz i planul de seciune.

Observaie:Clasificarea trunchiurilor de piramid se face dup


numrul de laturi ale poligonului de la baz (triunghiulare,
patrulatere,hexagonale), dup natura poligonului de la baz(regulat,
neregulat) i dup felul cum sunt feele laterale(trapeze isoscele sau
nu trunchi drept sau oblic).

56
Definiie: Distana dintre bazele trunchiului se numete nlimea
trunchiului. Ea poate fi calculat ca diferena dintre nlimea
piramidei din care provine trunchiul i nlimea piramidei noi
formate.

CORPURI ASEMENEA. RAPORTUL DE ASEMNARE.

n urma secionrii unei piramide cu un plan paralel cu baza se


obine o piramid de acelai vrf cu baza n planul de seciune
asemenea cu piramida iniial.

Definiie: Raportul a dou segmente omoloage corespunztor unei


perechi de corpuri asemenea se numete raportul de asemnare .

Definiie: Raportul ariilor a dou suprafee omoloage corespunztor


unei perechi de piramide asemenea este egal cu ptratul raportului de
asemnare.

11. SECIUNI AXIALE N CORPURILE CARE ADMIT AX


DE SIMETRIE

Definiie: Dou puncte Ai B sunt simetrice fa de un punct O, dac


O este mijlocul segmentului AB.
Definiie: Un punct O este centru de simetrie al unei figuri plane
dac orice punct al figurii are simetric fa de O tot un punct al
figurii.
Definiie: O figur geometric plan admite o ax de simetrie d dac
orice punct al figurii are simetric fa de dreapta d tot un punct al
figurii.

Axa de simetrie a unui corp este dreapta fa de care punctele unui


corp sunt simetrice.
Axa de simetrie a unei piramide regulate este dreapta ce trece prin
vrful piramidei i centrul bazei.

57
Axa de simetrie a unei prisme drepte i a unui trunchi este dreapta ce
trece prin centrele bazelor.

Definiie: Se numete seciune axial a unui corp, poligonul obinut


prin secionarea printr-un plan care conine axa de simetrie a
corpului.
Observaie: Seciunile axiale n poliedre sunt variabile ca form, iar
n corpurile de rotaie sunt congruente.
La prisme , planele care conin diagonalele se numesc seciuni
diagonale( sau plan diagonal).

Cap II PERPENDICULARITATE N SPAIU

12. Proiecii de puncte, drepte i segmente pe un plan

Definiie. Se numete proiecie a unui punct pe un plan,


piciorul perpendicularei duse din acel punct pe un plan.
Definiie: Se numete proiecie a unei figuri geometrice pe un
plan mulimea proieciilor punctelor acelei figuri pe plan.

Teorema 12.1 Proiecia unei drepte pe un plan este o dreapta


sau un punct.

58
Teorema12.2 Proiecia unui segment pe un plan este un
segment sau un punct.

Teorema 12.3. Proiecia mijlocului unui segment pe un plan


sau dreapt este mijlocul proieciei acelui segment pe planul
dat sau pe dreapta dat.

13. Unghiul unei drepte cu un plan

Definiie: Unghiul unei drepte cu un plan este unghiul fcut


de dreapt cu proiecia ei pe plan.

59
Teorema13.1. Unghiul unei drepte cu un plan este cel mai
mic dintre unghiurile formate de acea dreapt cu o dreapt
oarecare a planului.

Teorema 13.2. Lungimea proieciei unui segment pe un plan


este egal cu lungimea segmentului nmulit cu cosinusul
unghiului format de dreapta suport a segmentului cu planul.

AB= AB.cos u

Teorema 13.3. Aria proieciei unei figuri pe un plan este egal


cu aria figurii date nmulit cu cosinusul unghiului fcut de
figur cu planul. Aria ABC = Aria ABC cos u

14. Diedru. Unghi plan corespunztor unui diedru.

Definiie: Se numete diedru reuniunea a dou semiplane care


au aceeai frontier.
Definiie: Se numete unghi plan corespunztor unghiului
diedru, unghiul format de dou semidrepte coninute n feele
diedrului, i perpendiculare pe muchia diedrului n acelai
punct.
Teorema 14.1. Orice dou unghiuri plane corespunztoare
diedrului au aceeai msur.

60
Definiie: Msura unui diedru este msura unui unghi plan
corespunztor diedrului.
Definiie: Msura unghiului dintre dou plane secante este cea
mai mic dintre msurile diedrelor formate de aceste plane i
este egal cu msura unghiului format de dou drepte
perpendiculare respectiv pe planele date.

15. Plane perpendiculare

Definiie: Dou plane se numesc perpendiculare dac unul


dintre diedrele determinate de ele are msura de 90.
Teorema15.1 Dou plane secante sunt perpendiculare dac i
numai dac unul dintre plane conine o dreapt perpendicular
pe cellalt plan.
Teorema15.2 Dac dou plane sunt perpendiculare, proiecia
pe unul dintre plane a oricrui punct din cellalt plan aparine
dreptei de intersecie a planelor.

16. Teorema celor trei perpendiculare.

Teorema celor trei perpendiculare: Dac o dreapt d este


perpendicular pe un plan i prin piciorul ei ducem o
perpendicular f pe o dreapt g din acel plan, atunci dreapta
determinat de un punct de pe d i de intersecia celor dou
drepte din plan, este perpendicular pe dreapta g.
Prima reciproc a T. C.3. P: Dac dintr-un punct exterior
unui plan ducem perpendiculara pe un plan i perpendiculara
pe o dreapt din plan, atunci dreapta ce unete picioarele celor
dou perpendiculare este perpendicular pe dreapta dat din
plan.

61
A doua reciproc a T.C.3:P: Dac ntr-un punct al unei drepte
dintr-un plan se duc dou drepte perpendiculare pe ea, prima
exterioar planului i a doua coninut n plan atunci
perpendiculara dintr-un punct al primei drepte pe cea de-a doua
este perpendicular pe plan.

POLIEDRE I CORPURI ROTUNDE

PRISMA TRIUNGHIULAR REGULAT

A l = Pb h
At = Al + 2 Ab
V = Ab h

PRISMA PATRULATER
REGULAT

62
A l = Pb h
At = Al + 2 Ab
V = Ab h

CUBUL

a= muchia cubului
D = a 3
Al = 4 a 2

At = 6 a 2

V = a3

PARALELIPIPEDUL DREPTUNGHIC
a,b,c = dimensiunile
paralelipipedului
D = a2 + b2 + c2
Al = 2 ac + 2bc
At = 2 ab + 2bc + 2ca
V = a b c

63
TETRAEDRUL REGULAT

l =latura tetraedrului
l 6
h=
3
3l 2 3
Al =
4
4l 2 3
At =
4
3
l 2
V =
12

PIRAMIDA REGULAT

Pb a
=
p
A l
2
A t = A l + A b

A h
V = b

64
TRUNCHIUL DE PIRAMID REGULAT

(PB + Pb ) at
Al =
2
At = Al + AB + Ab
h
V = (AB + Ab + AB Ab )
3

CORPURI ROTUNDE

CILINDRUL CIRCULAR DREPT

Al = 2RG
At = 2RG (G + R )
V = R 2 H

65
CONUL CIRCULAR DREPT

A l = RG
At = R (G + R )
R 2 H
V =
3
G 2
= H 2
+ R2

TRUNCHIUL DE CON CIRCULAR DREPT

Al = G( R + r )
At = G( R + r ) + R 2 + r 2
h
V= (R2 + r 2 + R r)
3
G = h2 + (R r)2
2

66
SFERA

A = 4 R 2

4 R 3
V =
3

67
PROBLEME ENUNTURI SI REZOLVARI

CLASA A V A

1. O veveri aduce alune la vizuin n 14 minute, tiind c


ea fuge fr alune cu 3m/s i cu alune cu 2m/s, aflai
distana de la alun la vizuin.
Rezolvare:
Timpul ct fuge veveria este dat de raportul dintre lungimea
drumului i viteza cu care fuge:
Aadar, fie d distana de la alun la vizuin; Atunci
d d 5d
+ = 14 60 = 840 d = 1008m .
3 2 6

7
2. S se determine a 2007-a zecimal a fraciei .
11
7
Rezolvare: =0,(63); Rezult c a 2007-a zecimal este 6.
11

3. S se afle n N din egalitatea:


2 2 n -4=3(4+4+.42007)

Rezolvare : 2 2 n =4+ 3 4 + 3 4 2 + ... + 3 4 2007 . Din aproape n


aproape ,rezult 22n=42008 22n=24016 , rezult n=2008.

4. S se determine n ; n<10, astfel nct


9n + 4
2n 1
Rezolvare:
Pentru n=1 si n=9 => 9n + 4
2n 1

68
5. S se arate c N=44n+2 .53n+3 .34n + 28n+1 .53n .34n+1 este
divizibil cu 2006 n N*
Rezolvare:
N=(44)n 42 (34)n (53)n 53 + (28)n 2 (53)n (34)n 3 =
=28n 34n 53n (24 53+2 3)=
2 (10 3 + 3 )


=28n 34n53n = 2006 # 2006

6. S se arate c numrul
N=2005 2007 +2006 2008 +2007 2005 +2008 2006 nu e ptrat perfect.
Rezolvare : Evident u (2005 2007 ) = 5
u (2006 2008 ) = 6
u (2007 2005 ) = u (7 4*501+1 ) = u (71)
=7
u (2008 2006 ) = u (8 2006 ) = 4
u (N) = u (5+6+7+4) = 2 N nu e ptrat perfect.

7. S se afle ultima cifr a numrului


N = 1 2006 +2 2006 +3 2006 ++2005 2006 +2006 2006
Rezolvare : u (1 2006 ) = 1
u (2 2006 ) = 4
u (3 2006 ) = 9
u (4 2006 ) = 6
u (5 2006 ) = 5
u (6 2006 ) = 6
u (7 2006 ) = 9
u (8 2006 ) = 4
u (9 2006 ) = 1
u (10 2006 ) = 0
Ultima cifr a primelor zece numere este 5
u (N) = 5 * 400 + 1+ 4 + 9 + 6 +
5 +
6

69
= 0 + 31
u (N) = 1.

8. Sa se determine numrul natural a, pentru care:


3 2
27 a 9 a 3 a = 72917
Rezolvare: Relaia data este echivalenta cu:
3 2
33a 3 2 a 3 a = (36 )17
3 2
33a + 2 a + a = 3102 3a 3 + 2a 2 + a = 102
Pentru a = 3 327 + 29 + 3 = 102

9. Sa se arate ca numarul:
N = 2007 + 2008 ( 1+2+3+.+2006 ) 2 este cubul
unui numr natural.
Rezolvare:
N = 2007 + 2008 ( 2006 2007 ) : 2 2 =
= 2007 + 2008 2006 2007 =
= 2007 [ 1 + ( 2007+1 ) ( 2007-1 )] =
= 2007 ( 1 +20072 1 ) = 20073

10. Sa se compare numerele:

A=20062007+20072006 si B=20062006+20072007

Rezolvare: 2006=2005+1 , 2007=2006+1

A=200620062006+20072006=20062006+200520062006+20072006
B=20062006+200720072006=20062006+200620072006+20072006
Cum 2005<2006 si 20062006<20072006 =>A<B

70
11. Se considera numerele a1=4, a2=3a1=4, a3=3a2+42,........
a100=3a99+499,.....
a)Sa se determine a2007.
b)Sa se compare numerele a100 cu 3150
Rezolvare:
Se observa ca pentru orice numr natural n, avem
an=3an-1+4n-1=4 =>
a) a2007=42007
b)a100=4100=2200=(24)50
3150=(33)50 cum 24<33 =>a100<3150

12. S se rezolve ecuaia:


x-2004 x-2005 x-2006 x-2007
9 - 159 +79 + 79 = 2007
Rezolvare :
Ecuatia dat este echivalent cu:

9x-2007( 7+159-89+9)=2007=> 9x-2007 223=9223=> 9x-


2007
=91=> x-2007=1=>x=2008

13. Calculai ctul i restul mparirii numrului 972007


la 272005
Rezolvare :

D=IC+R R<I 972007 =(7+2)72005 72 =73 . 72005


2005 2005
+2497 =(342+1)7 +49272005
(1712+1)72005 +49272005 = 171272005 +72005
2005
+4927 =>
=>972007 = (171+49)272005 +72005
Ctul este 220
2005
Restul este 7.

71
14. Gsii toate numerele naturale ptrate perfecte mai
mici dect 2007 care la mprirea cu 45 dau restul 36.
Rezolvare :
Fie n un numr natural. Rezult (conform T impartiri cu rest)
c n=45c+36 , n<2007
=>9(5c+4)<2007=>5c+4 223 5c+4 trebuie sa fie patrat
perfect
=>5c+4 {2 ;3 ;7 ;8 ;12 ;13 ;}cum
n=3(5c+4)=>n {6 ;9 ;21 ;24 ;36 ;39}

15. mprind numrul 185 la un numar natural se obine


restul 15.Aflai mpritorul.
Rezolvare:
D=IC+R R<I
185=XC+15=>185-15=XC cum170=2517 si R<X adic
X>15
=>X poate fi 17 sau 217=34 sau 517=85 sau 2517=170
Aadar mpritorul aparine mulimi{17,34,85,170}.

16. Sa se arate c nu exist nici un numr natural n astfel


nct 20072007 = 7n +2007
Rezolvare :
20072007-2007=7n nu poate fi adevrat pentru nici un
numr natural n deoarece ultima cifr
a lui (20072007 - 2007)este U(20072007 ) -7=6 ; U(72007)=3 ; 13-
7=6, n timp ce U(7n ) poate fi 7,9,3,sau 1,n nici un caz 6.

17. Dac ab + bc + ac =264, unde a,b,c sunt cifre n baza 10


atunci suma a+b+c este egal cu .;
Rezolvare:
ab + bc + ac =264 11(a+b+c)=1124 a+b+c=24.

72
18. Aflai numrul natural n pentru care 4n+4n+4n+4n=256
Rezolvare : Este evident ca 4n+4n+4n+4n=4.
n n+1 2n+2 8
4 =4 =2 =256=2 . Deci se obtine ca 2n+2=8, adica n=3.

19. Dac ab6 + b8a + 7ab = 2007 atunci ab # 17.


Rezolvare:
100a+10b+6+100b+80+a+700+10a+b=2007,
rezult 111a+111b+786=2007
111(a+b)=1221, a+b=11
Pentru a=3 i b=8 se verific relaia dat.

20. S se arate c numrul n= (20072007-2007)(20062006-


2006)(20052005-2005) este divizibil cu 200.
Rezolvare:
Ultima cifr a numrului din prima parantez este 6, a celui din
a doua parantez este 0, i a celui din a treia parantez este 0,
rezult produsul n are ultimele dou cifre 00 i cum
20072007-2007 este divizibil cu 2 rezult c n este divizibil cu
200.

21. S se arate c numrul 1+3+5+7+......+2005+2007 este


ptrat perfect.
Rezolvare:
Notm cu s suma 1+3+5+7+..............+2005+2007 =S i tot cu
s suma 2007+2005+2003+.............+3+1=S, dup ce adunm
sumele rezult ca 2008+2008+...................+2008 = 2S rezult
S=2008.1004:2=10042.

22. S se arate c numrul N= 4 n +1 5 2 n +5 + 2 2 n + 4 25 n + 2 e


ptrat perfect, oricare ar fi n natural.
Rezolvare:
N= 2 2 n + 2 5 2 n +5 + 2 2 n + 4 5 2 n + 4 =

73
2 2 n + 2 5 2 n + 4 (5 + 4) = (2 n +1 5 n + 2 ) 2 3 2 ceea ce nseamn c este
ptrat perfect.

23. S se determine valoarea numrului natural a astfel


nct numrul N=4a2+2a.a+2a+1 s fie ptrat perfect.
Rezolvare:
Pentru a= 5 se obine N= 324=182.

24. O persoan mplinete n anul 2007 o vrst egal cu


dublul sumei cifrelor anului de natere. Aflai vrsta
persoanei i anul cnd s-a nscut.
Rezolvare:
19xy+2(1+9+x+y)=2007

1965+2(1+9+6+5)=2007. Aadar x=6 i y=5.

25. S se determine numerele de forma ab tiind c


7 ab + ba = abba : 11 + 1 .

Rezolvare:

7(10a+b)+10b+a=(1001a+110b):11+1

71a+17b=91a+10b+1 20a+1=7b a=1; b=3.

26. Suma a patruzeci i dou de numere naturale nenule


este 900. Artai c exist printre aceste numere cel puin
dou egale.
Rezolvare:
Dac considerm primele 42 de numere naturale consecutive
atunci 1+2+3+4+....42= (42.43):2=903, aadar putem avea n
loc de 4 pe 1.

74
27. ntr-o ncpere sunt 11 scaune cu trei respectiv patru
picioare. tiind c s-au folosit 40 de picioare metalice
pentru scaune, s se spun cte scaune cu trei respectiv
patru picioare s-au fcut.
Rezolvare:
Presupunem c la scaunele care au cte patru picioare le rupem
un picior n aa fel nct s avem numai scaune cu trei picioare,
n total un numr de 33 de picioare. Aadar din 40 scdem 33 i
obinem 7 adic cele 7 picioare rupte provin tot de la attea
scaune cu patru picioare. n rest scaunele cu trei picioare sunt
n numr de 11-7=4.

28. a) S se arate c numrul N=6+62+63+64+.....+6100 este


divizibil cu 7;
b) S se afle ultima cifr a lui 2007 2005 N .
Rezolvare:
a) N=6(1+6)+63(1+6)+........+699(1+6) este divizibil cu 7;
b) u(N)=u(20072005)u(N)=u(72005)u(N)=0.

29. Determinai numrul natural x din egalitatea:


[( )2
( )] 4
a) 310 35 : 81 + 3 9 3 : x = 3 25.
[ ]
b) 5 + 125 6 : (5 2 ) 7 5 3 x = 10 + 20 + 30 + ....1000
( ) ( )
Rezolvare: a) 3 25 4 : x = 3 25 x = 3 25 4 : 3 25 x = 4.
b)
505 x = 10(1 + 2 + 3 + 4 + .... + 100) 505 x = 10 (100 101) : 2
Rezult: x=100.

30. S se determine numrul natural n astfel nct


4 n +1 + 4 n
n +1
= 2 2007 .
2 + 3 2 n

Rezolvare:

75
2 2n+ 2 + 2 2n 5 2 2n
= 2 2007
= 2 2007 2 n = 2 2007 n = 2007
2 2n + 3 2n 5 2n

2 m 3 m +1 + 2
31. S se arate c pentru m, n N, fracia n +1 n se
7 8 + 8
simplific cu 10.
Rezolvare:
2 m 3 m +1 + 2 6 m 3 + 2
= . Cum ultima cifr a numrtorului i
7 n +1 8 n + 8 56 n 7 + 8
a numitorului este 0, rezult c fracia se simplific cu 10.

32. S se determine numerele naturale n astfel nct fracia


n2 + 3
s se simplifice cu 5.
n3 + n2 + 5
Rezolvare:

Fie 5| n+3 si 5| n+ n+5 => 5| (n+ n+5)-(n+3) =>5| n+3


Atunci n poate fi de forma 5K, 5K+1, 5K+2,5K+3, 5K+4

Se observ c numai pentru n = 5k+2 se obine c fracia dat


se poate simplifica cu 5 deoarece:
(5K+2) +2 = M5+8+2 # 5.

33. S se determine numrul natural n astfel nct fracia


n2 5
N.
n3
Rezolvare:
n2 5 4
= n +3+ N n 3 4 n {4,5,7}.
n3 n3

76
34. ntr-o urn sunt 6 bile albe, 8 bile roii i 12 bile negre.
Care este cel mai mic numr de extrageri pe care-l putem
face astfel nct s fim siguri c am extras cel puin 5 bile
de aceeai culoare.
Rezolvare: Presupunand ca extragerile se fac n aa mod nct
de fiecare dat extragem o bil de culoare diferit de cea
extras anterior(ex. alb, roie, neagr), dup 3 astfel de cicluri
de cte 4 bile de culori diferite, deci dup 12 extrageri, cea de-a
zecea extragere ne va aduce 5 bile care vor avea aceeai
culoare.
Deci numrul minim este 13.

35. S se arate c numrul n= 11112222 n baza zece se


poate scrie ca un produs de dou numere naturale
consecutive.
Rezolvare:
n=1111*104+2222=1111(104+2)=1111(104-
1+3)=1111(9999+3)=1111(3.3333+3)=3333(3333+1)
=3333*3334

36. S se arate c numrul :

16057 1 1 1
N= + + ++ este ptrat
2007 1 2 2 3 2006 2007
perfect.

Rezolvare :

77
1 1 1 1 1 1
+ ++ = - + -
1 2 2 3 2006 2007 1 2 2
1 1 1
+ ... + =
3 2006 2007
1 2007 1 2006
=1 = =
2007 2007 2007

16057 + 2006 18063


= = 9 = 32
2007 2007

N= 3 2

37. S se rezolve ecuaia : xy + x - 4y = 2007

Rezolvare : Ecuaia dat este echivalent cu (x-4) (y+1) =


2003

x-4=1 i y+1=2003 x=5, y=2002 sau


x-4=2003 i y+1=1 x=2007, y=0 sau
x-4= -1 i y+1= -2003 x=3, y= -2004 sau
x-4= -2003 i y+1= -1 x- -1999, y= -2

78
CLASA A VI A
ALGEBRA

1. Numerele a,b,c verific simultan condiiile:


1
1
a) media aritmetic a numerelor a,b,c este 0,4(1)
3
b) a,b,c sunt direct proporionale cu 0,(3), 0,0(3), 0,00(3).
S se afle a,b,c.
Rezolvare:
37 111
a+b+c= 3 3 =
90 30
a b c a+b+c
= = = = 10
0, (3) 0,0(3) 0,00(3) 3 3 3
+ +
9 90 900
a=30; b=300; c=3000.

6n
2. S se determine restul mpririi lui 4 la 33.
Rezolvare:
4 6 = 4 ( 5 +1 ) = 4 ( 5 k +1 ) = ( 4 5 ) k 4 = 1024 4 = (1023 + 1) k 4
n n
k

Rezult c restul mpririi este 4.


Observaie: ntodeauna, un numr de forma (a+1)n va fi un
multiplu de a la care se adaug 1.

2 2007 x
3. Aflai x Q din proporia: 1004
= .
4 0,5
Rezolvare: Se rescrie 41004 ca putere a lui 2, se fac
simplificarile si rezulta x=1.

4. S se determine numerele ntregi x,y dac


x2+xy-2x-2y=3.
Rezolvare:

79
Ecuaia dat este echivalent cu (x+y)(x-2)=3
x+y=1 i x-2=3 x=5, y=-4;
x+y=3 i x-2=1 x=3, y=-2;
x+y=-1 i x-2=-3 x=-1,y=0;
x+y=-3 i x-2=-1 x=1,y=-4.

5. S se rezolve n numere naturale, ecuaia:


xy-5=3x+2y-13.
Rezolvare:
xy-5=3x+2y-13 xy-3x=2y-13+5
x(y-3)=2y-
2 y 8 2( y 3) 2 2
8 x = = = 2 N, y 3
y 3 y 3 y 3
y 3 D2
y-3=1 y=4; x=0;
y-3=-1 y=2; x=4;
y-3=2 y=5; x=1;
y-3=-2 y=1; x=3.

a 2 2a + 3
6. S se determine a Z astfel nct Z .
a+2
Rezolvare:

a2 2a +3 a(a +2) 4a +3 4a +3 4(a +2) +8+3


= = a+ = a+ =
a +2 a +2 a +2 a +2
11
a 4+ Z a +211
a +2
a +2 D11 = {11,1,1,11}

a+2=-11 => a=-13


a+2=-1 => a=-3
a+2=1 => a=-1

80
a+2=-11 => a=9.

x 3
7. S se rezolve n Z ecuaia: = 2.
3 y
x 3
Rezolvare: = 2 xy 9 = 6 y ( x 6) y = 9
3 y
x-6=1 i y=9 x=7,y=9
x-6=-1 i y=-9 x=5,y=-9
x-6=9 i y=1 x=15,y=1
x-6=-9 i y=-1 x=-3,y=-1
x-6=3 i y=3 x=9,y=3
x-6=-3 i y=-3 x=3,y=-3

8. Sa se calculeze :
1 1 1 1
+ + + ... +
1 2 2 3 3 4 2006 2007
Rezolvare :
Suma dat este egal cu
1 1 1 1 1 1 1 1
+ + + ... + =
1 2 2 3 3 4 2006 2007
1 2006
=1 =
2007 2007
S-a folosit formula :
1 n +1 n 1 1
= = .
n(n + 1) n(n + 1) n n + 1

9. S se determine numerele ntregi pozitive a si b care


satisfac condiiile a+b=165 si ctul mpririi lui a la b este
10.
Rezolvare:

81
a=10b+r , 0 r < b
a+b=165=> 10b+r+b=165 => 11b+r= 11 15 => r este divizibil
cu 11=> r=11x=> 11(b+x)= 11 15 => b+x=15=> x=1,
deoarece r=11x<b => r=11 => b=14 => a=151 .

2x 4
10. S se determine mulimea A = x Z
x3
Rezolvare :
2x 4 2
= 2+ Z x 3 2 x 3 D2 = { 2,1,1,2}
x3 x 3

x 3 = 2 x = 1

x 3 = 1 x = 2

x 3 =1 x = 4

x 3 = 2 x = 5.

11. S se rezolve n N ecuaia: 4 x + 8 y + 64 z = 192


Rezolvare:

2 2 x + 23 y + 2 6 z = 26 3 : 26 2 2 x 6 + 23 y 6 + 26 z 6 = 3

Cum 2 x 6 < 1,
3y 6 < 1
6z 6 < 1 2x 6 = 0 x = 3
3y 6 = 0 y = 2
6z 6 = 0 z = 1
82
12. S se rezolve n numere ntregi pozitive ecuaia :
3x+5y2=189.
Rezolvare:
Cum 3x i 189 se divid cu 3 => 5y2 e divizibil cu 3 =>
y=3a => 3x+5*9a2=189 :3 =>
x+15 a2=63 => x=3 b => 3 b+15 a2=63 :3 =>
b+5 a2=21 => b=21, a=0 => x=63, y=0
b=1 , a=2 => x=3, y=6.

13. S se determine numerele ntregi pozitive a si b care


satisfac condiiile a+b=165 si ctul mpririi lui a la b este
10.
Rezolvare:
a=10b+r , 0 r < b
a+b=165=> 10b+r+b=165 => 11b+r= 11 15 => r este divizibil
cu 11=> r=11x=> 11(b+x)= 11 15 => b+x=15=> x=1,
deoarece r=11x<b => r=11 => b=14 => a=151 .

14. a) S se calculeze suma A=1+11+111++ 111


 ... 111

2007
b) S se arate c A este divizibil cu 9
c) S se calculeze B=3+33+333++ 33  ...

33
2007
Rezolvare :

1
a) A= (9+99+999++999)
9
10 1 10 2 1 10 3 1 10 2007 1
A= + + + ... +
9 9 9 9

83
1 1
(10 + 102 + ... + 102007 2007) = (111
0 2007) =
...111
9 9 
2007

=
1
= (111...
1109103)
9 
2003

b) Cum suma cifrelor din care este compus A este


2003 1 +0+9+1+0+3=2016 care este divizibil cu 9 A# 9
1
c) B=3A= ( 111 ...11091033

)
3  2008

GEOMETRIE

1. Msura unui unghi al unui triunghi isoscel este de


1100.
a) Msurile celorlalte dou unghiuri sunt de
;
b) Suma msurilor tuturor unghiurilor exterioare
triunghiului este de ..;
c) Msura unghiului format de bisectoarea unghiului de
1100 cu latura opus este de .;
Rezolvare:
a) Celelalte dou unghiuri sunt obligatoriu de 350;
b) Pentru fiecare unghi al triunghiului exist dou unghiuri
exterioare congruente prin urmare suma msurilor tuturor
unghiurilor exterioare triunghiului este de 36002= 7200.
c) 1800-(550+350)=900.

84
2. n dreptunghiul ABCD cu AB=10 cm i BC =18 cm se
ia un punct M pe AB i N pe BC astfel nct AM este de
trei ori mai mic dect NC i NC este de dou ori mai mare
dect BN. S se afle aria triunghiului MND.
Rezolvare:

Din datele problemei, rezult NC=3AM i NC=2BN Cum


BC=18, rezult NC=12 i AM=4. Aadar AMND=AABCD-AMBD-
ANCD-AAMD=216-24-72-36=84.

3. n triunghiul ABC se ia D un punct pe latura BC astfel


nct AD este congruent cu DC. Dac perimetrul lui ABC
este de 46 cm i al lui ABD este de 30 cm se cere lungimea
lui AC.
Rezolvare:
AC= PABC-PABD=46-30=16 cm.

4. n paralelogramul ABCD se ia pe diagonala AC


punctele E i F astfel nct AE=EF=FC. S se arate c
patrulaterul BEDF este paralelogram i c AABCD=3ABEDF.
Rezolvare.

85
Din congruena triunghiurilor ABE i DFC (L.U.L), rezult BE
i DF sunt paralele i congruente, rezult BEDF paralelogram.
AABCD=6ABEF=3ABEDF
5. Fie ABCD un trapez avnd bazele AD, i BC, cu
BC>AD i AD=10cm. Paralela prin A la CD intersecteaz
BC n E. tiind c perimetrul triunghiului ABE este 24 s
se afle perimetrul trapezului.

Rezolvare:

Cum AE este congruent cu DC, rezult c perimetrul


trapezului este PABCD= PABE+2AD=24+20=44 cm.

6. Fie ABCD un paralelogram cu AB=8 cm i cu

m( B )=1500. Se cere msura unghiului A i distana de la


B la AD. Dac AD=12 cm, aflai aria paralelogramului.=

Rezolvare:

86
Cum m( B )=1500, rezult c unghiul A are 300 iar BE,
unde E este piciorul perpendicularei dus din B pe AD este
jumtate din AB. Rezult BE este de 4 cm.
AABCD=AD*BE=12*4=48cm2.

7. n rombul ABCD se duc DPAB i BQCD. tiind c


msura unghiului A este de 600 s se arate c:
a) triunghiul ABD este echilateral;
b) patrulaterul BPDQ este dreptunghi;
c) s se afle ct la sut reprezint aria patrulaterului BPDQ
din aria lui ABCD.

Rezolvare:
Triunghiul ABD este isoscel cu un unghi de 600, rezult c
triunghiul ABD este echilateral. Patrulaterul BPDQ este
dreptunghi deoarece are dou laturi opuse paralele i
congruente.
Cum msura unghiului A este de 600, atunci msura unghiului
ADP este de 300 ceeace nseamn c AP este jumtate din AD.
Cum AB este congruent cu AD i DPAB chiar n mijlocul lui
AB, rezult AABCD=4AAPD. Rezult, ABPDQ=50%AABCD.

8) n patrulaterul ABCD, AC este perpendicular cu BD ca


n figura alturat. C
Dac AC=13cm i BD=8cm s se afle aria lui ABCD.

87
C

B
D

Rezolvare:

Cum DB i AC sunt perpendiculare, rezult


BD AC 104
AABCD=AABD+ACBD= = = 52 cm2
2 2

9. n figura alturat se dau AC=6,BD=8cm, AE=18,CB=12


cm i A,C,B puncte coliniare.
Dac EAAC, BDBC s se afle aria triunghiului ECD.

88
E
D

C
A B

Rezolvare:
AECD=AABDE-AAEC-ADBC=234-54-48=132cm2.

11. Fie ABC un triunghi oarecare. Prin mijlocul lui M al


laturii AB se duce paralela la AC care intersecteaz pe BC
n N. Prin B se duce paralela la AC care intersecteaz pe
AN n D.
a) Aflai natura patrulaterului ABDC.
b) Dac aria triunghiului AMN=12 cm2 aflai aria
patrulaterului ABDC.

Rezolvare:
MN este linie mijlocie n triunghiul ABD respectiv n BAC,
atunci N este mijlocul lui AD i al lui BC, rezult c
patrulaterul ABCD este paralelogram.
n triunghiul ANM, NM este median, atunci AANM=ANBM,
n triunghiul ABD, BN este median, atunci AABN=ABND ,
n triunghiul ABC, AN este median iar n triunghiul BDC, DN
este median, rezult AABDC=4AABN=4*24=96cm2.

89
CLASA A VII A
ALGEBRA

1. S se rezolve ecuaia n .
( x 3) + y 2 y = 8
2 2

Rezolvare:
x-3 + y(y-2) =8 => y{0,1,2,3,4}
y=0 => x=1;
y=1 => x=10;
y=2 => x=11;
y=3 => x=8;
y=4 => x=3.

2. S se determine cifrele consecutive a i b astfel nct


2 2
aa + bb aa bb = 8833.
Rezolvare:
Relaia dat este echivalent cu (aa bb ) 2 = 8833 aa bb
Cum a i b sunt cifre consecutive, rezult:
(aa bb ) 2 = 112 121 = 8833 (10a + a)(10b + b)
11a 11b8833 121 112 a b = 8712 : 121 a b = 72
a=9, b=8 sau a=8, =9.

3. S se determine numerele ntregi n astfel nct


n2 3
Z
n3
Rezolvare:
n 2 3 (n 3) (n 2 + 3n + 9) + 24 24
= = n 2 + 3n + 9 + Z
n3 n3 n3
n-3 24 n-3 ={ 1, 2, 3, 4, 6, 8, 12, 24}

90
=> n={-21,-9,-5,-3,0,1,2,3,4,5,6,7,8,9,11,15,27} .

4. S se rezolve ecuaia n N: xy-3x+3y=2016.


Rezolvare:
Ecuaia dat este echivalent cu (x+3)(y-3)=2007; Cum
2007=32.223 rezult urmtoarele situaii:
x+3=9 i y-3=223 rezult: x=6, y=226;
x+3=227 i y-3=9 rezult: x=220, y=12;
x+3=3 i y-3=669 rezult: x=0, y=672;
x+3=669 i y-3=3 rezult: x=666, y=6;
x+3=2007 i y-3=1 rezult: x=2004, y=4.

1 1 1
5. S se arate c 1 + + + .... + 2007 .
2 3 2007
Rezolvare: Cum
1 1 1 1 1
1 , ,........,
2007 2 2007 2006 2007

1 1 1 1
1+ + + .... + +
2 3 2007 2007
1 1 1
+ + + .... +
2002 2007 2007

2007
= = 2007 .
2007

6. S se determine numerele ntregi a i b astfel nct


4 6 + 14 = a 2 + b 3;
Rezolvare:

91
Ridicm la puterea a doua expresia dat:
4 6 + 14 = 2a + 2 6ab + 3b ;
2 2

Din egalarea termenilor asemenea ntre ei rezult : ab=2 i


2a2+3b2=14 rezult: a=1 i b=2.

7. Care numr este mai mare


A = 16 15sauB = 18 13 ?
Rezolvare: Prin ridicare la puterea a doua se obine: 0<A2<B2
rezult A este mai mic dect B.

1 1
8. Dac a =7, s se calculeze a4 + 4 .
a a
Rezolvare:
1 2
Ridicm la puterea a doua relaia dat: ( a ) =49,
a
1
a2+ =51 procednd analog se obine
a2
1 1
a 4 + 4 = 512 2 a 4 + 4 = 2599 .
a a

9. S se arate c numrul
N = 2007 + 2007 2 + 2007 3... + 2007 2007 este divizibil cu 2006 .
Rezolvare :
2007 2007 1 2007 2008 2007
Evident N = 2007 = Z
2007 1 2006
fracia se simplic cu 2006 N # 2006 .

10. S se rezolve n Z ecuaia :


( x 5 )2 + y 2 + 2 y =8

Rezolvare :

92
x 5 + y( y + 2) = 8
y = 1 x 5 = 5 x = 10
y = 2 x5 = 0 x = 5

11. S se rezolve n mulimea numerelor ntregi ecuaia


5 x 2 + 6 xy + 5 y 2 = 400
Rezolvare:
Ecuaia dat este echivalent cu 4(x+y)2 +(x-y)2=400.
Notm x + y = a si
2
x y = b 4a 2 + b 2 = 400 b = 4b1 4a 2 + 4 2 b1 = 400
a 2 + 4b 2 1 = 100 a = 4a1 4a 21 + b 21 = 25
b1 este impar
b1 {1,3,5}
b1 = 3 a1 = 2 (1)
si
b1 = 5 a1 = 0 (2 )

x= y =8 x+ y =0
x y = 12 x y = 20
(1) (2)
x = 10 x = 10
y = 2 y = 10

12. Aflai X din


1 1 1
X.3 2008 = (3 2008 1) : (1+ + 2 + ....... + 2007 )
3 3 3
Rezolvare:
1 1 1 3 3 2008 1
1+ + + ........ + 2007 = 2008 , dup formula
3 3 3 2 3
X n +1 1
1 + X + X + ......... + X n =
X 1 93
2 3 2008 2
X 3 2008 = [3 2008 1] 2008 X =
3 3 1 3
2a 3
13. S se calculeze: unde a = 7 11 4 7
3a
Rezolvare:
11 + 3 11 3
11 4 7 = = 7 2 a = 2
2 2

2 2 3 (2 2 3 )( 3 + 2 )
= = 2 6 + 4 3 6 = 6 +1
3 2 3 2

a
14. tiind ca: = 3 1 s se calculeze partea ntreag a
b
a + b
numrului
a b
Rezolvare:
a
= 3 + 1 a = 3 + 1, b = 1
( ) = 3+ 2
3 +1 +1 3 +1+1
=
b ( 3 + 1) 1 3 + 2 3 +11

=
5+2 3
=
(5 + 2 3 )(3 2 3 ) =
3+ 2 3 9 12
15 10 3 + 6 3 12 3 4 3 4 3 3
= = =
3 3 3
4 3 3
=1
3

94
15. Se d numarul x = 6 2 5 6 + 2 5
a) S se arate ca x = 4
b) S se calculeze (X+2)2007
Rezolvare:

a. (1 5 ) (1 + 5 ) =
X= = 1 5 1 + 5 = 1 + 5 1 5 = 2 x = 4

b. x = 2 x + 2 = 0 ( x + 2) 2007 = 0.

a 66b
16. Daca = 2007 , s se calculeze .
b a 223 9b

Rezolvare:
66b 66 66 1
a = b 2007 = = =
b 2007 223 9b 223 3 9 660 10

17. S se calculeze suma


S= 2 + 2 2 + 2 3 + .......... + 2 2007 .

Rezolvare:

S= ( 2 + 23 + 2 5 + .......... .. + 2 2007 )+
+ 22 + 24 + .......... .. + 2 2006 =

= (
2 1 + 2 + 2 2 + .......... .. + 2 1003 + )
+ 2 + 2 + 2 + .......... ... + 2
2 3 1003
+11 =
(
= 1 + 2 + 2 2 + .......... ... + 2 1003 )( 2 +1 1 = )
= [ (2 1004
) 1]( )
2 + 1 1.
95
Am adugat i am sczut 1.

18. Calculai:
( )
E = 4 + 2 3 + 7 4 3 + 2 68 351 + 2 68 : 350
Rezolvare:
4+2 42
4+2 3 = + = 3 +1
2 2
7 +1 7 1
74 3 = = 2 3
2 2
(2 ) (3 )
4 17 3 17
<0
( )
E = 3 + 1 + 2 3 + 268 + 351 + 268 : 350 =
= 3 + 351 : 350 = 3 + 3 = 6.

14 6 5 + 6 2 5
19. Determinai n Z astfel nct Z.
n
Rezolvare:

(3 5 )2
+ 5 1
2
(=
)
3 5 + 5 1
=
n n
3 5 + 5 1 2
= = Z n { 2,1,1,2}
n n

96
20. Dac x 2 + y 2 = 12 xy , x,y R, x,y>0
x+ y
S se calculeze : a )
x y
b)
x
y
c ) partea ntreag a numrului
x
.
y
Rezolvare :
a) ( x + y ) 2 = x 2 + y 2 + 2 xy = 14 xy

( x y ) 2 = x 2 + y 2 2 xy = 10 xy
x+ y 14 7 35
= = =
x y 10 5 5
b) 5 x + 5 y = 35 x 35 y ( 35 5) x = y (5 + 35 )
x 5 + 35
=
y 35 5
x (5 + 35 ) 2
25 + 35 + 10 35 60 + 10 35
c) = = = =
y 35 25 10 10
= 6 + 35
x
y = 6 + 5 = 11

21. Sa se arate ca a= 1 + 3 + 5 + ...2007 Q b=


1 3 5 7 ..... 2007

2
tiind ca: 1+3+5++2007= 1004 , => a

97
2
Cum 2003 135..2007 iar 2003 , nu apartine
produsului=> b .

x2 +1
22 Determinai x Z pentru care fracia Z
x+2
Rezolvare:
x 2 + 1 x ( x + 2) 2( x + 2) + 5 5
= = x2+ Z
x+2 x+2 x+2
x + 2 5 x + 2 D 5 = { 1, 5}

x+2=-1 x=-3
x+2=-5 x=-7
x+2=1 x=-1
x+2=5 x=3 .

x 2x + y 1
23. Dac = 15% atunci = ..............% .
y 3x + 2 y + 1
Rezolvare:
x 15 3
= 15% = = x = 3; y = 20
y 100 20
2x + y 1 2 3 + 20 1 25 50
= = = = 50%
3 x + 2 y + 1 3 3 + 2 20 + 1 50 100

x 3
24. S se rezolve n Z ecuaia: = 2.
3 y
x 3
Rezolvare: = 2 xy 9 = 6 y ( x 6) y = 9
3 y
x-6=1 i y=9 x=7,y=9
x-6=-1 i y=-9 x=5,y=-9
x-6=9 i y=1 x=15,y=1

98
x-6=-9 i y=-1 x=-3,y=-1
x-6=3 i y=3 x=9,y=3
x-6=-3 i y=-3 x=3,y=-3.

25. S se determine numerele ntregi x,y dac x2+xy-2x-2y=3.


Rezolvare:
Ecuaia dat este echivalent cu (x+y)(x-2)=3
x+y=1 i x-2=3 x=5, y=-4; x+y=3 i x-2=1 x=3, y=-2;
x+y=-1 i x-2=-3 x=-1,y=0; x+y=-3 i x-2=-1 x=1,y=-
4.

26. S se rezolve n numere naturale, ecuaia: xy-5=3x+2y-13.


Rezolvare:
xy-5=3x+2y-13 xy-3x=2y-13+5
x(y-3)=2y-
2 y 8 2( y 3) 2 2
8 x = = = 2 N, y 3
y 3 y 3 y 3
y 3 D2
y-3=1 y=4; x=0; y-3=-1 y=2; x=4;
y-3=2 y=5; x=1; y-3=-2 y=1; x=3.

27. Se dau numerele x,y,z, astfel nct x este 20% din y, iar y este 30%
din z. Se cere:
x y z
a) S se calculeze suma + + ;
y z x
b) Dac 50x+10y+z=350 atunci s se afle x, y, z.
Rezolvare:
x 20 1 y 30 3 z 100
a) = = ; = = ; =
y 100 5 z 100 10 x 6
x y z 1 3 100 103
+ + = + + = .
y z x 5 10 6 6

99
y 1 3z 3 3
b) Cum x = = = z i y = z 50x+10y+z=350
5 5 10 50 10
3z+3z+z=350 7z=350 z=50
x=3 i y=15.

28. Numerele naturale a i b sunt direct proporionale cu 4 i 10.


a) Ce procent din numrul a reprezint numrul b?
b) Media aritmetic a numerelor a i b este 28. Aflai numerele a i b.
Rezolvare:
a b 10 250
a) = b= a= a b = 250% din a ;
4 10 4 100
a+b a b 56 28
b) = 28 a + b = 56 = = = = 4.
2 4 10 14 7

29. Fie n , s se afle restul mpririi lui N= 58 n +5 la 25.


Rezolvare:
N = (5 4 n ) 2 5 5 = [(5 4 n ) 2 1 + 1] 5 5 =
= [(5 4 n 1)(5 4 n + 1) + 1] 5 5 =
= [(52 n 1)(52 n + 1)(54 n + 1) + 1] 55
Cum (5n-1) 4, (5n+1) 2, (52n+1) 2,(54n+1) 2, rezult c
restul mpririi lui N la 25 este restul mpririi lui 55 la 25 care
este 21.

100
GEOMETRIE

1. Fie triunghiul ABC cu D AB, E BC astfel nct AADF=8, AAFC=20,


ACEF=15. Se cere aria patrulaterului BDFE.
Rezolvare:
Se tie c n orice triunghi ABC cu E BC exist egalitatea :
AABE BE
= , atunci
AAEC EC
vom nota pentru simplitate
cu A1=AADF, A2=AAFC,
A3=ACEF, A4=ADFE,
A5=ABDE;

A2 AF A1 20 8
= = = A4 = 6, De asemenea
A3 FE A4 15 A4
A2 + A3 EC A4 + A3 20 + 15 6 + 15
= = = A5 = 21,
A1 + A4 + A5 BE A5 8 + 6 + A5 A5
ABDFE=A4+A5=6+21=27.

2. Un trapez isoscel are lungimea diagonalelor de 5 cm i suma


lungimilor bazelor de 6 cm. S se afle aria trapezului.

101
Rezolvare:
Fie BM ||AC, M (DC , CM AB, DM=DC+AB=6 i
BM AC=5 BDM isoscel cu BE DM, BE nlime
2
DM
BE2=BM2-EM2 BE= 5 2 =4
2
( AB + CD ) DE DM DE 64
AABCD = = = ADMB = = 12 .
2 2 2

3. n triunghiul dreptunghic ABC, m( A )=900, se ia pe latura BC


punctual F la 2 cm de vrful C i pe AC se ia punctual E astfel nct
CF CA 1
= = . S se calculeze perimetrul triunghiului ABC.
CE BC 3
Rezolvare:

Cum FC =2 cm, rezult din relaia dat c CE=32=6 cm,


relaie care mai poate fi scris i sub forma :
CF CE
= i cum unghiul C este comun, rezult c triunghiul
CA BC
CFE este asemenea cu triunghiul CAB(cazul L.U.L.) CFE
este dreptunghic n F. Din Teorema lui Pitagora aplicat n

102
triunghiul CFE FE2=CE2-FC2
FE= 36 4 = 32 = 4 2 .
Din CFE ~ CAB
CA 1 FE
= = AB = 3 FE = 12 2 cm i BC=3CA
BC 3 AB
Din triunghiul ABC dreptunghic, rezult: BC2=AB2+AC2
(3AC)2=( 12 2 )2+AC2
AC=6 cm i BC= 18 cm
PABC=AB+AC+BC= 12 2 +18+6=12( 2 +2) cm.

4. n trapezul dreptunghic ABCD, AB||CD, AB=12 cm, CD=6 cm,


CE 1
AD=8 cm se ia pe diagonala AC punctul E astfel nct = . Din E
CA 4
se duce o perpendicular pe AC astfel nct ea intersecteaz pe CD n F.
Se cere:
a) perimetrul trapezului ABCD;
b) s se arate c CEF ~ CDA i s se calculeze lungimea
segmentului EF;
c) s se calculeze distana de la E la AB.
Rezolvare:

103
D F N C

A B
M
Prin punctul E se duce o perpendicular pe bazele trapezului pe
care le intersecteaz n MAB i NCD. Cum CD=6 i
AB=12 rezult c triunghiul CAB este isoscel (nlimea din C
cade chiar la mijlocul lui AB). Din T. lui Pitagora aplicat n
ADC CB AC=10.
Atunci PABCD= AB+BC+CD+AD=6+8+12+10=36.
b) CEF ~ CDA deoarece sunt triunghiuri dreptunghice cu
EF CE
unghiul C comun, = (*)
DA DC
CE 1 CA 5 10
Cum = CE = = Din (*) EF= .
CA 4 4 2 3
NE CE NE 1
c) Din NE|| AD = =
AD CA 8 4
d(E,AB)=EM=MN-NE=8-2=6 .

5. n triunghiul dreptunghic ABC, m( A )=900, iar AC=12 cm, BC=13


cm. Se cere lungimea bisectoarei BD.
Rezolvare:

104
Aplicnd teorema lui Pitagora n triunghiul ABC, obinem
AB=5 cm. Din Teorema bisectoarei, rezult:
AD AB 5 AD 5
= = =
DC BC 13 AD + DC 5 + 13
AD 5 5 12 10
= AD = =
12 18 18 3
10 5
BD = ( ) 2 + 5 2 = 13 .
3 3

AD 1
6. n ABC, D AB, F AD, E AC astfel nct = i AF=2cm,
DB 4
DE BC, FE DC. Se cere lungimea lui AB.

Rezolvare:
Aplicnd de mai multe ori
teorema lui Thales,
1 AD AE AF
= = = =
4 DB EC FD
2

FD
FD=8 AD=10
10 1
= DB=40 AB
DB 4
=AD+DB=50cm.

105
7. n trapezul ABCD, cu BCAD, BC=12 se duce o paralel EF la
bazele trapezului, care trece prin intersecia diagonalelor O, astfel nct
AE 1 A
= . Se cere AEO .
AB 3 A AOD
Rezolvare:

A D

F
E
O
B C
12

EOBC Din Teorema fundamental a asemnrii


AE EO 1 EO
= = EO = 4
AB BC 3 12

AE 1 AE 1
= =
AB AE 2 EB 2
AD OD EA AD 1
OAD ~ OBC = = = AD = 6
BC OB EB 12 2
A AEO EO 4 2
= = = .
AAOD AD 6 3

8. Fie ABC un triunghi i fie AD o bisectoare astfel nct la o treime


fa de vrf, prin punctul P de pe bisectoare se duce o paralel MN
la BC.
Din N AC se duce o paralel la AB care intersecteaz pe AD in Q
si pe BC in R.
Dac AB=18, BC=16 si AC=6, aflai perimetrul triunghiului QDR.
Rezolvare:

106
AB BC 18 16
BD = = = 12. Analog DC=4 Cum MN||BC,
AB + AC 24
rezult:

AP 1 MP PN 1 MP PN 4
= = = = = MP = 4, PN = .
AD 3 BD DC 3 12 4 3
MN=MP+PN
=
4 16 16 16 20
4+ = BR = MN = RD = BD BR = 12 =
3 3 3 3 3

Cum PN||RD atunci triunghiurile PQN i DQR sun asemenea:


QN PN 4 QN + QR 4 + 20
= = =
QR RD 20 QR 20
BM 24 20 BM
= QR =
QR 20 24

AM AP 1
= = ; AM = 6 BM = 12 QR = 10.
AB AD 3
Din formula lungimii bisectoarei,
bc 18 6
AD 2 = [(b + c) 2 a 2 ] = [(18 + 6) 2 16 2 =
(b + c) 2
(18 + 6) 2

107
60. AD= 60 = 2 15
AP 1 2 15 2 15 4 15
= AP = PD = 2 15 =
AD 3 3 3 3
QN PQ 4 1 PQ + QD 6 PD 6
= = = = =
QR QD 20 5 QD 5 QD 5
5 4 15 10 15
QD = =
36 9
10 15 20 150 + 10 15
PQRD=QR+QD+RD= 10+ + = .
9 3 9
9. S se afle aria triunghiului ABC, dac se cunosc AC=2
cm i m( B )=0,1(6) m( A ) i m( C ) = 0,8(3) m( A ) ).
Rezolvare:
15 1 75 5
0,1(6)= = i 0,8(3)= = . Cum
90 6 90 6
m( A ) + m( B ) + m(C ) = 180 0
1 5
m( A ) + m( A ) + m( A ) = 180 0
6 6
m( A ) = 90 m( B ) = 15 0 , m(C ) = 75 0
0

Aadar, triunghiul ABC este dreptunghic n A . Cum AC se


opune unui unghi de 150,rezult:
BC=4AC=8 i AB= 64 4 = 60 = 2 15.
AB AC
AABC= = 2 15cm 2 .
2

10. n triunghiul dreptunghic ABC, cu m( A ) = 90 0 , AB=6 cm, CD=9


cm unde AD este perpendicular pe BC, se duce prin mijlocul M al lui
AC o perpendicular pe AC care intersecteaz pe BC n P, iar
AD PM= {N } . Se cere lungimea segmentelor BD, AC, AN, AD.

108
Rezolvare:

Din Teorema nlimii n triunghiul ABC rezult:


AD 2 = BD DC = 9 BD,
Din Teorema lui Pitagora n triunghiul ABD rezult:

AB 2 = AD 2 + BD 2 36 = BD 2 + 9 BD
BD = 3 cm
AC 2 = BC 2 AB 2 AC = 6 3 .
AB AC BC
MAN = =
AM MN AN
Cum ABC ~
6 6 3 12
= = AN = 6 3 , AD = 3 3
3 3 9 AN
AM MC
P este mijlocul lui BC. Aadar: BP=PC=6,
MP AB
DP=6-3=3.

109
DBA ~
DPN
3 3 3 6
= = DN = 3 3 , NP = 6 , MP = 3, MN = 9 .
DP DN NP

11. Fie ABCD un paralelogram cu baza BC=15 cm n care


diagonalele BD=18 cm formeaz un unghi de 300 cu AC=10
cm; S se calculeze perimetrul paralelogramului.
Rezolvare:
Fie AE nlimea paralelogramului dus din A pe BC;
AC BD sin 30 0 BC AE
AABCD = =
2 2
18 10
= 15 AE AE = 6;
2
Aplicm T. lui Pitagora n triunghiul dreptunghic AEC;
EC = AC 2 AE 2 = 100 36 = 64 = 8; BE=15-8=7;
Aplicm t. lui Pitagora n triunghiul dreptunghic ABE:
AB = 36 + 49 = 85

P = (30 + 2 85 )cm .

12. Pe ptratul ABCD de latur 12cm se ia punctual M pe


BC astfel nct BM =3cm si N pe CD astfel nct DN=1cm.
Se cere sinusul unghiului MAN.
Rezolvare:
Aplicm Teorema lui Pitagora n triunghiurile dreptunghice
ABM si AND .
AM= . AN= .
AABM=18, AMCN=18, AADN=48=>AAMN= AABCD-18-18-48=>
AAMN=144-84=60.

110
AM AN sin(< MAN )
Pe de alt parte ,AAMN= =>
2
10 221
sin MA N =
221

13. Fie ABCD un trapez oarecare cu bazele AB, CD si O


intersectia diagonalelor AC si BD. Paralela prin O la baze
taie laturile neparalele in M si N. Demonstrati ca :
a) OM = ON;
b) Daca PQ este linia mijlocie a trapezului, atunci
MN . PQ = AB . CD
Rezolvare:
a)Deoarece MO AB rezulta ca triunghiurile DOM si DBA
sunt asemenea ceea ce conduce la : MO / AB = DO / DB.
In mod analog vom avea egalitatea ON / AB = CO / AC.
Folosind faptul ca ABCD este trapez, deci triunghiurile AOB si
COD sunt asemenea, adica DO / DB = CO / AC, va rezulta ca
MO / AB = NO / AB.Deci MO = NO.
b)Aceleasi asemanari de triunghiuri ne dau urmatoarele
egalitati, in care am folosit faptul ca OM = ON:

MN / AB = 2 MO/ AB = 2 DO / BD si
MN / CD = 2 NO / CD = 2 BO / CD, relatii care prin adunare
ne dau
MN / AB + MN / CD = 2( DO / BD + BO / BD) =
2 BD / BD = 2.

111
Prin impartire la 2 si aducere la numitor comun in relatia de
mai sus se obtine
MN(AB + CD) / 2 = AB . CD, adica exact ceea ce trebuia
demonstrat, pentru ca linia mijlocie PQ este exact semi-suma
bazelor.

14. n triunghiul oarecare ABC se tie:


AB=10 BC=14 AC=12. Se cere:
a) Aria triungiului
b) nlimea AD

Rezolvare:
a) A ABC = p( p a)( p b)( p c) unde
10 + 14 + 12
P= =18
2
A= 18(18 10)(18 14)(18 12) =24 6
BC h
b) A ABC =
2
14 h 24 6
24 6 = h=
2 7

15. n figura alturat , AOB este un sfert de cerc cu raza


de 8 cm i perimetrul suprafeei MPNO este de 20 cm. Dac
MP este paralel cu OB i PN paralel cu OA se cere s se afle
perimetrul figurii AMNBPA.

112
Rezolvare:
Patrulaterul OMPN este dreptunghi, rezult 2(OM+ON)=20,
aadar OM+ON=10.
Pe de alt parte diagonalele dreptunghiului sunt congruente,
rezult MN=OP=8 cm.(OP este raz). Rezult,
AM+BN=AO+OB-(OM+ON)=8+8-10=6.
Cum lungimea arcului AB este
2R 2 8
l A B = = = 4 PAMNBPA= AM+MN+NB+lAB=
4 4
6+8+4=14+4.

113
CLASA A VIII -A
ALGEBRA

1
1. Fie f : N R , f (n ) =
n +1 + n
S se calculeze:
f (1) + f (2 ) + f (3) + ... + f (2007 )
Rezolvare:
1 n +1 n
= = n +1 n
n +1 + n n +1 n
f (1) + f (2 ) + ... + f (2007 ) = 2 1 + 3 2 + 4 3 +
+ ... + 2006 2005 + 2007 2006 = 2007 1

2. S se determine x i y din:

Rezolvare:
Relatia data este echivalenta cu :

x+1=0
x+y-3=0 => x=-1; y=4

3. Sa se simplifice fractia:
x 4 2 x 3 3x 2 + 8 x 4
E(x)=
x 4 + 2x3 x 2 2x

114
Rezolvare :
x 4 x 3 x 3 + x 3x 2 + 3x + 4 x 4
E(x)= =
x 3 ( x + 2) x( x + 2)
x 3 ( x 1) x( x 2 1) 3 x( x 1) + 4( x 1)
x( x + 2)( x 2 1)
( x 1)( x 3 x( x + 1) 3 x + 4) ( x 1)( x 3 x 2 4 x + 4)
= =
x( x + 2)( x 2 1) x( x + 2)( x 1)( x + 1)
x 2 ( x 1) 4( x 1) ( x 1)( x 2)( x + 2)
= = =
x( x + 2)( x + 1) x( x + 2)( x + 1)
( x 1)( x 2)
= .
x( x + 1)

4. Fie f: AR, f(x)=x-3;


a) S se reprezinte graficul funciei f cnd A=R, A=[0;);
A=(-3;2] .
b) S se determine mulimea A={n N | n 20, f(n) e ptrat
perfect};
f (n + 2)
c) Artai c fracia e o fracie ireductibil,
f (n + 1)
n N ;
d) S se rezolve n R inecuaia : f(7-2x) f(3x+2);
e) S se determine cel mai mic numr natural x pentru care
exist n N astfel nct f(3x+4)=7n;
2x 4
f) S se rezolve ecuaia: f + f ( x + 5) = 1.
3
g) Aflai numerele reale a i b cu proprietatea 2a-3b=4 i c
punctual A(a,b) aparine graficului lui f;
h) tiind c A(m,n) este punctul de intersecie al graficului
lui f cu graficul funciei g:RR, s se determine m i n.

115
i) S se calculeze aria triunghiului determinat de axele de
coordonate i graficul lui f.
Rezolvare: a)

b) f(n) e ptrat perfect n-3 = k2 n{3,4,7,12,19}.


f (n + 2) n 1
c) Presupunem prin absurd c fracia = este
f (n + 1) n 2
reductibil adic exist un divizor comun diferit de 1
d = (n 1, n 2) ,
d 1 d n 1, d n 2 d n 1 n + 2 d 1 d = 1 .
Contradicie; rezult c f este ireductibil.
d) f(7-2x) f(3x+2) 7-2x-3 3x+2-3 5x 5
x 1 x [1;+) .
e) f(3x+4)=7n 3x+4-3=7n 3x+1=7n ; Pentru n=3
x=114 este cel mai mic numr natural pentru care este
adevrat relaia dat;

116
f)
2x 4
f + f ( x + 5) = 1.
3
2x 4
3 + x + 5 3 = 1 2 x 4 + 3x = 6 x = 2 ;
3
g) A(a,b) Gf f(a)=b a-3=b
a b = 3 (2)
b = 2; a = 5 ;
2a 3b = 4
y = x 3
h) x = 7; y = 4 ; A(7,4).
y = 2 x + 18
i) Triunghiul format de axele de coordonate i graficul lui f este
33 9
dreptunghic A= = = 4,5 .
2 2

5. a) S se determine funcia f:RR, f(x)=ax+b, a,b R cu


proprietatea f(x-2)=3x-2f(2)+3;
b) S se reprezinte graficul lui f,
c) S se rezolve ecuaia: 3f(x-1) -2f(x)=x+7.
Rezolvare:
a) Din f(x-2)=3x-2f(2)+3 a(x-2)+b=3x-2(2a+b) +3 (a-
3)x+2a+3b-3=0 a=3 i b=-1; f(x)=3x-1.

117
b)

2
x
O1 2
-1

c) 3[3(x-1)-1]-1-2(3x-1)=x+7 x=2.

6. Fie f:RR , g:RR, f(x)= 3 2 x 3 ,


g(x)= 3 x + 2 2 .
a) S se calculeze: f(1)+g(1)+f(-2)+g(-2);
b) S se rezolve inecuaia: f(x) g(x);
c) S se determine m R astfel
nct f( m + 2 )=g( m + 2 3 ).

Rezolvare:
a) f(1)+g(1)+f(-2)+g(-2)= 2 3 3 .

118
3 2 x 3 3x + 2 2 (3 2 3 ) x 2 2 + 3
b) 2 2+ 3 (3 2 + 3 ) (2 2 + 3 )
x x
3 2 3 18 3
3+ 6
x .
3
c)
f( m + 2 )=g( m + 2 3 )
2 2+ 3
3 2 (m + 2 ) 3 = 3 (m + 2 3 ) + 2 2 m =
3 2 3

3+ 6
m= .
3

7. Dac f:A{0; 3 ;4}, f(x)=3x-2 atunci A=.


2 3+2
Rezolvare: f(a)=3a-2=0 a= ; f(a)= 3 a= ;
3 3
f(a)=4 a=2
2 3+2
A={ , ,2}.
3 3

8. Se consider funcia f:RR, f(x)=6x-5.


a) Soluia ecuaiei f(x-2)=1 este egal cu...............-;
b) Soluiile ecuaiei f(x)=x2 sunt;
c) Suma f(1)+f(2)+f(3)+.f(50) este egal cu ..;
d) Numrul a R pentru care f(f(a)) =37 este.;
e) Soluiile n Z ale ecuaiei f2(x)-8f(x)+7=0 sunt ;
f) Lungimea segmentului AB unde A(m,1) i B(2,n) sunt
puncte ale graficului lui f este.
Rezolvare:
a) f(x-2)=1 6(x-2)-5=1 x=3;

119
b) f(x)=x2 x2-6x+5=0 x=1 sau x=5;
c) f(1)+f(2)+f(3)+.f(50)= 6(1+2+3+4++50)-550= 3
5051-550=50(153-5)=7400.
d) f(f(a)) =37 6(6a-5)-5=37 a=2.
e) f2(x)-8f(x)+7=0 f(x)=1 6x-5=1 x=1 sau
f(x)=5 6x-5=7 x=2.
f) A(m,1) Gf f(m)=1 6m-5=1 m=1;
B(2,n) Gf f(2)=n 12-5=n n=7. A(1,1), B(2,7)
AB= (2 1) 2 + (7 1) 2 = 37

9. Fie f:RR, cu proprietatea f(xy)=f(x)+f(y). S se


calculeze f(2007).
Rezolvare:
Pentru x=y=0 f(0)=2f(0) f(0)=0; Fie y=0 i x 0
f(0)=f(x)+f(0) f(x)=0, x R f(2007)=0.

10. Trei numere naturale consecutive a,b,c au proprietate


ab+bc+ca=146; S se calculeze suma a+b+c.
Rezolvare:
Fie cele trei numere de forma a, a+1, a+2 Atunci
ab+bc+ca=146 a(a+1)+(a+1)(a+2)+(a+2)a=146
2 2
3a +6a+2=146 3a +6a-144=0 a=6.
Numerele sunt 6,7,8 iar suma lor este 21.

11. S se determine numerele ntregi a, astfel nct numrul


a 2 + 7a + 10 s fie ptratul unui alt numr ntreg.
Rezolvare:
Z , a.i, a 2 + 7 a + 10 = k 2 4a 2 + 28a + 40 = 4k 2
Fie k
4a 2 + 28a + 49 9 = 4k 2
( 2a + 7 ) 2 ( 2k ) 2 = 3 2 ( 2a + 7 2k ) ( 2a + 7 + 2 k ) = 9

120
Cum divizorii lui 9 sunt { 1,3,9}
2 a + 7 2 k = 1 9 1 9 3 3

2 a + 7 + 2 k = 9 1 9 1 3 3

4a + 14 = 10,10,10,10. 10.6. 6

a = 1,1,6,6,2,5

a { 6,5,2,1} .

12. S se arate c numrul:

116 + 2007 116 2007


este raional.
2
Rezolvare:
Notm numrul dat cu k . Prin ridicare la puterea a doua
rezult:
116 (116 + 2007 )(116 2007 = k 2
116 116 2 2007 = k 2
116 11449 = k 2 k 2 = 9 k { 3,3}.
Aadar, k este numr raional.

121
GEOMETRIE

1. S se arate c suma distanelor de la un punct oarecare


din interiorul unui tetraedru regulat la feele sale este egal
cu nlimea tetraedrului.
Rezolvare :
Fie P un punct n interiorul tetraedrului ; => VVABC =
VPVAB+VPABC+VPVBC+VPVAC = =1/3.S (d1+d2+d3+d4), unde :
- S aria unei fee ;
- di distana de la P la una din fee.
Cum : VVABC=(S*V0)/3 =>
=> V0= d1+d2+d3+d4

2. Fie VABCD o piramid patrulater regulat avnd


muchia bazei de 4 i muchia lateral de 2 5. a) S se
calculeze aria seciunii axiale VBD.
b) S se afle sin( BVD ).
c) S se determine poziia unui punct P, interior diagonalei
BD , astfel nct ducnd din acest punct perpendicularele
PM respective PN pe dou muchii laterale opuse , s se
formeze un segment MN avnd lungimea cea mai mic i s
se calculeze aceasta.
Rezolvare:

122
BD VO 4 2 2 3
a) ABVD = = =4 6
2 2
VB VD sin BVD
ABVD = =
2
2 5 2 5 sin BVD
b)
2
8 6 2 6
sin BVD = =
20 5
c) Singurul punct pentru care MN este este minim este atunci
cnd punctul P coincide cu O, centrul bazei. Se ia P un puct
arbitrar pe BD i se duc perpendicularele PM pe VB i PN pe
VD. Atunci patrulaterul VMPN este inscriptibil deoarece suma
unghiurilor M i N este 1800. Rezult
sin BVD = sin MP N (1) i PM N PVN
PN
sin PM N = sin PVN = (2)
VP

PM MN sin PM N PM PN sin MP N
APMN = = (3)
2 2
PN
Din (1) i (3) rezult: PM MN = PM PN sin BVD
VP
MN = VP sin BVD
MN este minim cnd VP este minim, aadar VP coincide cu
VO, nlimea piramidei.
2 6 12 2
MN = 2 3 = .
5 5

123
3. Fie VABCD o piramid patrulater regulat avnd
muchia lateral VA=4 5 cm i muchia bazei AB=8 cm. Se
cere:
a) aria total i volumul piramidei;
b) msura unghiului format de o muchie lateral cu planul
bazei;
c) msura unghiului format de o fa lateral cu planul
bazei;
d) sinusul unghiului format de dou fee laterale opuse;
e) Distana de la centrul bazei la o fa lateral;
f) lungimea segmentului determinat de centrele de greutate
a dou fee laterale opuse;
g) sinusul unghiului determinat de dou fee alturate;
Rezolvare:
a) Fie M mijlocul lui AB; Din Teorema lui Pitagora aplicat n
VAM obinem VM= VA 2 AM 2 = (4 5 ) 2 4 2 = 8 cm;
Pb a p 4 AB VM
Al= = = 128cm 2 .
2 2
At=Al+Ab=128+64=192 cm2
VO este nlime n triunghiul echilateral VMN VO
l 3 8 3
= = = 4 3 cm;
2 2
A h 64 VO 64 4 3 256 3 3
V= b = = = cm ;
3 3 3 3

124
b) VA formeaz cu (ABC) unghiul VA O . Cum AO
AC 8 2
= = =4 2
2 2
VO 4 3 6
tg (VA O) = = = ;
AO 4 2 2
c) Cum VMN este echilateral m(VN O) = 60 0 ;
d) Unghiul format de planele VAB i VDC este unghiul
determinat de nlimile celor dou triunghiuri i are msura de
3
600, sin 600= ;
2
e) Vom calcula distana de la centrul bazei, O, la planul VDC.
Fie P VN astfel nct OP VN (1)
Evident c CD (VON) OP CD (2)

125
Din (1) i (2) OP (VDC)
VO ON 4 3 4
d(O,(VDC))=OP = = = 2 3 cm;
VN 8
f) Unind centrele de greutate a dou fee opuse se formeaz
dou triunghiuri asemenea n care raportul de asemnare este
2
egal cu . Fie G1G2 segmentul determinat de centrele de
3
greutate
G1G2 2 16
= G1G2 = cm.
MN 3 3
g) Fie AR VD i CR VD. Cum triunghiurile VAD i VDC
sunt isoscele AR i CR sunt congruente triunghiul ARC
este isoscel;
Cum VD (ARC) i OR (ARC) OR VD OR este
nlime n triunghiul ARC;
VO OD 4 3 4 2 4 30
n VOD dreptunghic, OR = = =
VD 4 5 5
AC OR 8 2 4 30 32 15
AARC = = = ; Pe de alt parte,
2 25 5
AR RC sin R
AARC= (*)
2
Mai rmne s aflm lungimea segmentelor AR i RC care se
afl din triunghiul isoscel VDC, scriind aria sa n dou
moduri:
CD VN VD CR 8 8 4 5 CR 16 5
AVDC = = = CR = = AR ;
2 2 2 2 5

32 15 (16 5 ) 2 sin R 15
Din relaia (*) = sin R = ;
5 5 2
2
4
Observaii:

126
O alt cerin a problemei putea fi determinarea msurii
unghiului dintre dreptele AC i VD care se determin
uor din punctual g) unde s-a artat c VD (ARC)
VD AC, rezultnd c dreptele fac un unghi de 900
ntre ele;
Deesemenea se poate cere determinarea unui punct Q i
VQ
a raportului , astfel nct aria triunghiului AQC s
QD
fie minim, aceasta avnd loc atunci cnd Q coincide cu
R, deoarece atunci distana OQ =OR este minim;
VQ=VR se afl din triunghiul dreptunghic VOR.

4. n prisma patrulater regulat ABCDABCD, cu muchia


bazei de 6 cm, se ia M respectiv N mijloacele muchiilor CC
respectiv DD. tiind c dreptele BM i CN formeaz ntre
ele un unghi de 600, s se afle muchia lateral.
Rezolvare:

CN|| MD triunghiul MBD este isoscel cu msura


unghiului M de 600, rezult c este chiar echilateral.

BD= 6 2 =MB=MD, Din triunghiul dreptunghic


BCM MC=6.

127
Aadar, lungimea muchiei laterale este CC=12.

5. n cubul ABCDABCD, cu muchia de 4 cm se cere


distana de la punctul A la planul ABC.
Rezolvare:
Se duce AP perpendicular pe AB.
Evident din BC AB, BC A' B
( A' AB) ( A' BC ) AP ( A' BC )
AA' AB
d(A, (ABC))=AP= =2 2.
A' B

6. Un paralelipiped dreptunghic are dimensiunile direct


proporionale cu numerele 2 , 8 ,3 i volumul de 648 2 .
S se afle lungimea diagonalelor paralelipipedului.
Rezolvare:

Fie a,b,c dimensiunile paralelipipedului, atunci:


a b c
= = = k a= 2 k , b= 8 k, c=3k.
2 8 3
V=abc; 648 2 =12k3 k= 3 2 a=6, b=12, c= 9 2 .
Atunci lungimea diagonalei este:

128
D = a 2 + b 2 + c 2 = 3 38 .

7. n cubul ABCDABCD cu muchi de 8 cm se ia M


mijlocul lui CC. S se determine cosinusul unghiului
format de planele ABM i ABC.
Rezolvare:

Se va aplica formula ariei proieciei unui triunghi pe un plan:


S=S.cos (*),unde S este aria proieciei, S este aria
triunghiului care se proiecteaz.
AABC=AABMcos
AB=8 2 ,
AABC=32,
Se duce o paralel prim M la BC care este perpendicular pe
BB n punctul N(N este mijlocul lui BB), iar din N se duce o
perpendicular pe AB n P. Conform teoremei celor trei
perpendiculare rezult c MP este perpendicular pe AB.
Aadar triunghiul BPN este asemenea cu BBA.

129
Rezult PN=2 2 . Cum MN este 8, atunci cu T lui Pitagora
obinem: MP2=MN2+PN2
A' B MP 8 2 6 2
MP=6 2 . AABM= = = 48 Din relaia (*)
2 2
rezult
32 2
cos= = .
48 3

8. n paralelipipedul dreptunghic ABCDABCD se


cunosc muchiile bazei AB=2 5 si BC=4 5 , iar aria sa
total este 70. Se cere tangenta unghiului fcut de AO cu
planul BDB unde O este centrul bazei de sus.

Rezolvare:
A t =2(ab+ac+bc)
Din A t =70 nlimea paralelipipedului este 5 .
Ducem AE BD, E BD, AE nlime n ABD, AE=4, O-
centrul bazei ABCD AEO dreptunghic n E
AE 4
tg(OA,(BDB))=tg(A O' E) =
EO' 3

130
PROBLEME PROPUSE

Operaii cu numere ntregi

1)(5 23) ( 11 + 13 + 2 14 );
2 )36 : ( 2 ) + 4 ( 9 ) 45 : ( 15) + 3 ( 10 ) : ( 15);
3)5 ( 6) 4 ( 2) + 6 (+ 7 ) 3 ( 15);
5
( )
4 )5 3 58 : ( 25) 2 5 35 : ( 6 ) + (16 ) : ( 8) ;
5 3 3

5)24 : {[30 + ( 6 ) ( 5)] 60 + 2} + 15 : [( 3) ( 1)];


[
6 )( 4 ) ( 8) 16 4 : 8 5 ;
2 4
]( ) 2

(
7 ) 5 30 5 20 ) ( 5) ( 5) ( 5) ( 5)
4 60 40 30 70
;
8)(11 1)(16 1) : (2 3 5) ;
2 2 2

9 )( 2 )
2007
{
: 2 2005 10 2 2 ( 4 ) : 4 4 2 ; [ 5
]}
{ [
10 ) 2 5 (3 2 ) : 2 2 + 2 6 2 2 5 ;
2
]} ( )
2

11)1 + 2 + 3 + 4 + ... + 2006 + 2007;


12 )2 + 4 + 6 + ... + 2004 + 2006;
13)1 + 3 + 5 + 7 + ... + 2005 + 2007;
14 )2 + 2 2 + 2 3 + ... + 2 2006 + 2 2007 ;
(
15) 2 2007 2 2006 2 2005 : ( 8) ; ) 668

(
16 ) 2 333 + 2 333 3 222 : ( 9 ) ; ) 111

(
17 )2 2007 2 2006 + 2 2005 + ... + 2 + 1 ; )
18)(9 3 ) (8
3 9
4 2 4 ;4 8
)( 4 2
)
19 )( 3) { 5 + 3 [( 6 ) (+ 2 ) + ( 5) ( 6) + ( 2)] ( 3)};
20 ) 12 + 12 {13 13 [ 5 ( 2 ) ( 3)] 6};

131
[
21)( 3)
11 12
] : [(3 ) ( 3) ] ( 4) ( 8) : ( 2) ;
12 10 9 12 7 2

22)( 2) : [16 : 2 : ( 10 + 8) 2 ( 2) ];
100 25 4 26 8 62

23){( 18) : ( 6 ) ( 2) ( 3) [20 + ( 4) ( 5)]} ( 57 );


[
24)( 2) ( 2) + ( 2)
46 45 44
]: [7 ( 2) ]; 44

25)( 1) 1 + ( 1) 2 + ( 1) 3 + ( 1) 4 + ... + ( 1)
0 1 2 3 2005
2006 +
+ ( 1)
2006
2007;
[ 2
( ) (
26)( 2) 2 2 2 1 : 2 2 + ( 2) + 2 1 ; )] [ 2
]
27 )10 11 10 + 11 10 11 10 + ... + 11 10 11 10 + 11 1;
10 9 8 7 2

(
28)2007 2 2007 2 2 2007 2 2006 2 2005 ... 21 2 0 ; )
29)5 2008
5 2007
5 2006
19 25 1003
;

30)S se compare numerele:


a)36 cu 32 ; b)7100 cu 7200 ; c)1023 cu 523 ;
d)330 cu 245 ; e)248 cu 332 ; f)251 cu 334 ;
g)2 cu 5 -5 ; h)(-3)75 cu (-2)125;
107 46 45
i)25100 cu 12549;
2
j)(-2)40 cu (-16)10; k)(54 )5 cu 5 2 ; l)(-2)71 cu (-3)71.

132
Mulimea numerelor raionale
0
1 2007
1) 0, (7 ) : + 2, (3) + ;
49 2006
1
2
1
2) 1, (3) (0,5) : 3 + 0, (6) : ;
2

3 2

3) (9 : 0,15 + 1 : 0,2) : 0,01;


15
4 ) 0,0(6 ) + 1,28 2[0,125 0,08(3)];
64
3
1 7 2 10
5) (0,01) : (0,7 ) + 2;
2

100 10 7
6) [0,4 3, (3) 1] : 0, (5) + 0,1;
1 4 3 1 2
3 : 3 + : 3
9 9 31 4 7
7) ;
3 3 4 1
6 :1 3 :
5 8 5 2
8)[1, (6) + 0,25] : 1,91(6 )

3 5 22 3 11 7 3
10

9) 2 : 2,75 4 1 + 1 ;
4 8 4 4
2 7 4
10 ) + + ;
3 5 15
3 13 1
11) + + ;
7 14 2

12 ) 23,56 + 5 63,64 0,56 + 0,04;

133
2 1 14
13) 2 1, (6) + ;
3 2 4
2
14) 2,5 6,7 + 2, (3) + + 3,2;
15
2 7 3 2
15) 2 + + + ;
3 12 4 6
3 2 15
16) 3 8 ;
7 5 18
5 18 38
17 ) ;
36 19 45
25
18) ( 0,2 ) ( 1,4 ) ;
49
1 17
2
2 36
19 ) ;
19 51
1
81 40
3 5 6
20) : : ;
16 8 4

134
9
21)( 4,5) : ( 0,7 ) : ;
14
6
3 5 3 26
22 ) : ;
4 4
14 15
1 1
23) ;
49 7
83 14 95
5 5 5
24 ) : ;
7 7 7

16 2 8
3 5 2

25) : ;
81 3 27
5
9 19 3 20 27 12 27 89
26 ) : : ;
25 5 125 125
3 2 2 5 7
27 ) 2 ;
4 3 3 6 12
1 1 1
+
2 3 4
28) ;
5 25
:
6 26
1 1 15 7 11
29 ) 2 + [ 3,2 (4,8 5)] + 0,2.
3 2 11 22 13

135
RAPOARTE I PROPORII

x 3 3x y
1. Dac = atunci =
y 5 x+ y

2. Numerele a,b,c sunt invers proporionale cu numerele


a+b
0,(3);1/5 respectiv 52. Atunci =
bc

3. Suma a trei numere este 680. Dac se mrete primul cu


50% din el, al doilea se micoreaz cu 25% din el,iar al treilea
se micoreaz cu 5, numerele devin egale.Ct la sut reprezint
primul numr din al treilea?

a2 b+3
4. Dac = , atunci suma a2+b2 este egala cu
3b 2+ a

5. Artai c 212+210+20 este ptrat perfect.

6. Fie a,b,c,dN,b>c. tiind c (a;b)(c;d)=(2;4) i


(a;b) (c;d)=(0,6)
atunci media aritmetic a numerelor a,b,c,d este

x yx
7. Dac = 20% atunci = ........
y 3x + y

a b b c
8. Dac = i = i a2+b2+c2=16500 atunci media
2 5 10 7
aritmetic a numerelor a,b,c este

136
9. Dac A={xN*| 22x+3<612} atunci probabilitatea ca un
numr din A s fie par este

x 5x + 7 y
10. Dac = 60% atunci =
y 9x 5 y

11. Numerele pozitive x,y,z sunt invers proporionale cu


1 1 1
; ; .Ct la sut reprezint numrul cel mai mic din
3 4 5
numrul cel mai mare?

x+ y x
12 .Dac = 5 atunci =
yx y

2100 299 2 2
99 98
= => x=
13. x 3

14. Dac 30% din lungimea unui drum reprezint 12 km,atunci


drumul are o lungime dekm.

x y
15. Dac = =>y reprezint %din x
5 3
x reprezint % din y

Dac x=85 =>y=

a b b c
16. Dac = si = , a+b+c=210,atunci a= b=
2 5 10 7
c=

137
a b b c
17. Dac = si = si a2+b2+c2=16500,atunci media
2 5 10 7
aritmetic a numerelor a,b,c este

18. Numerele a,b,c sunt invers proporionale cu numerele 0,2;


1
0,(3); . tiind c suma numerelor e 160 s se afle:
2
a) a,b,c;
b) ct la sut reprezint a din c.

19.Suma a doua numere este 120. tiind ca unul din ele este cu
25% din cellalt, s se afle:
a)cele doua numere;
b)raportul dintre cel mai mic si cel mai mare .
x 2 3x + 4 y x2
20. = => = ; 2 =
y 5 y y

x 0,3 x+ y x2 y2
21. = => = ; = ...
y 0,5 y y2

x x
22. = 5%, atunci =
y 5x + y

23. Dac x,y,z sunt direct proporionale cu 0,5; 0,(3); 0,(6) i


2x-3y+4z=12, atunci x-y+z=

2 5x
23. 30% din = 4 => x=
3 + 2x

138
ISTORICUL NOIUNILOR MATEMATICE

Sec. 18 .e.n. mesopotamienii creeaz primele tabele de


nmulire;
sec. 6 .e.n. este cunoscut asemnarea triunghiurilor
de ctre Thales;
Sec. 5 .e.n. pitagorienii introduc noiunile de numr
prim, numr compus, numere relativ prime, numere prime
perfecte;
Sec. 4 .e.n.
Aristotel (384-322 .e.n) filozof grec a introdus
noiunile de perimetru, teorem, silogism.
Sec. 3 .e.n.
Matematicianul grec Euclid(330-275 .e.n ) cel care a
ntemeiat celebra coal din Alexandria (n 323 .e.n)
a introdus noiunile de semidreapt, tangent la o curb,
puterea unui punct fa de un cerc sau sfer, sau
denumirile de paralelogram, poliedru, prism, tetraedru.
A enunat teorema catetei i a nlimii pentru un triunghi
dreptunghic i a demonstrat concurena mediatoarelor
unui triunghi;
n prima carte din Elementele lui Euclid este
cunoscut teorema mpririi cu rest i algoritmul lui
Euclid pentru aflarea c.m.m.d.c. a dou numere ntregi
Apolonius din Perga(262-200 .e.n), unul din cei mai
mari geometri ai antichitii introduce pentru prima dat
denumirile pentru conice, de elips, hiperbol, parabol
i noiunile de focare, normale i definete omotetia i

139
inversiunea i d o aproximare exact a lui cu patru
zecimale.
este dat aria triunghiului n funcie de laturi sau n
funcie de raza cercului nscris i semiperimetru;
Eratostene din Cyrene(275-195 .e.n) introduce
metoda de determinare a tuturor numerelor prime mai
mici dect un numr dat, metod cunoscut sub numele
de Ciurul lui Eratostene
85-168 matematicianul grec Ptolemeu prezint n
cartea sa Almagest, pe lng vaste cunotine de
astronomie i trigonometrie i diviziunea cercului n 360
de pri congruente i exprimarea acestora n fracii
sexagesimale.
Sec. 3 s-a dat formularea teoremei celor trei
perpendiculare de ctre Pappos; acesta a mai dat i
definiia conicelor precum i teorema despre volumul
corpurilor de rotaie
Sec. 7
sunt cunoscute regulile de trei direct i invers de
ctre Bragmagupta, matematician indian;
Arhimede(287-212 .e.n) precursor al calculului
integral, a determinat aria i volumul elipsoidului de
rotaie i ale hiperboloidului de rotaie cu pnze.
1202- Leonardo Fibonacci (1170-1240) matematician
italian introduce notaia pentru fracia ordinar;
1228- Fibonacci introduce denumirea pentru numrul
zero, precum i sistemul de numeraie zecimal. Tot prin
opera sa Liber abaci sunt introduse pentru dat n
Europa numerele negative, fiind interpretate ca datorii;
1150- este descris extragerea rdcinii ptrate i a celei
cubice n cartea Lilavati a matematicianului indian
Bhaskara(1114-1185), tot el prezint i operaiile de
nmulire i mprire cu numere negative;

140
1515- rezolvarea ecuaiilor de gradul al treilea cu o
necunoscut de ctre Scipio del Fero, iar mai trziu de
Niccolo Tartaglia n 1530, i pe acelea de gradul al
patrulea de Ludovico Ferrari n 1545. Acestea au fost
fcute cunoscute abia n 1545 de ctre Girolamo
Cardano(1502-1576) n lucrrile sale, dei promisese
autorilor lor s nu le divulge;
1591-matematicianul francez Francois Viete(1540-
1603) introduce formulele cunoscute sub numele de
relaiile lui Viete;
1614- inventarea logaritmilor naturali de ctre John
Neper(1550-1617);
1637- este introdus noiunea de variabil de ctre
Rene Descartes(1596-1650), cel care a introdus literele
alfabetului latin pentru notaii i a folosit coordonatele
carteziene (definite dup numele su), reducnd
problemele de geometrie la probleme de algebr;
1640- este introdus denumirea pentru cicloid de ctre
Galileo Galilei (1564-1642);
1654- nceputul crerii teoriei probabilitilor datorat
corespondenei dintre Pierre Fermat(1601-1665) i
Blaise Pascal(1623-1662) i dezvoltarea combinatoricii
odat cu apariia lucrrii lui Pascal, Combinaiones;
1656- matematicianul englez John Wallis(1616-1703)
1 1
introduce simbolul cu notaiile = , = 0 i a
0
denumirilor de interpolare respectiv mantis
1670- este determinat semnul sinusului i desenat
sinusoida respectiv secantoida de ctre John Wallis);
1678- este dat teorema lui Ceva de ctre Ceva
Giovani(1648-1734);
1679- n Varia opera mathematica aprut postum, a
lui Pierre Fermat(1601-1665), a fost dat Marea

141
teorem a lui Fermat, reguli de integrare, definiia
derivatei.
1692- este scris primul manual de calcul integral de
ctre matematicianul elveian Jean Bernoulli(1667-
1748) Lectiones mathematicae de methodo
integralium aliisque, tiprit abia n 1742 i de
asemenea a mai scris un manual de calcul diferenial,
descoperit abia n 1920.
Regula lui lHospital este dat de ctre Jean Bernoulli
lui Guillaume de lHospital pe care acesta o public n
1696;
1690- este propus denumirea de integral de ctre
Jacques Bernoulli(1654-1705)
1692- sunt descoperite proprietile spiralei logaritmice
(Jacques Bernoulli)
1694- este descoperit curba numit lemniscat,
caracterizat de inegalitatea
(1+x)n 1+nx (Jacques Bernoulli);
1696-1697- introducerea calculului variaional, punerea
problemei izoperimetrelor de ctre Jean Bernoulli.
1705- este dat Legea numerelor mari de ctre
Jacques Bernoulli;
1711- realizarea dezvoltrii n serie a funciilor ex, sinx,
cosx, arcsinx, de ctre matematicianul englez Isaac
Newton(1642-1727) cel care a pus bazele calculului
diferenial i integral concomitent cu Gottfried
Leibniz(1646-1716);
1729- este demonstrat existena rdcinilor complexe
n numr par a unei ecuaii algebrice cu coeficieni reali
de ctre Mac Laurin Colin(1698-1746;
1731- utilizarea sistemului de axe perpendiculare pentru
a determina poziia unui obiect n funcie de cele trei
coordonate;

142
1733- crearea trigonometriei sferoidale de ctre Alexis
Clairaut(1713-1765);
1735- Matematicianul elveian Leonhard Euler(1707-
1783) introduce i calculeaz constanta
1 1 1
e= lim(1 + + + ... + ln n) =0,577215..., n;
2 3 n
1739- introducerea conceptului de integral curbilinie
de ctre Alexis Clairaut;
1746- relaia lui Stewart este demonstrat de Mathew
Stewart dup ce n prealabil ea i fusese comunicat de
ctre Robert Simson n 1735;
1747
este enunat problema celor trei corpuri de ctre
Clairaut;
introducerea metodei multiplicatorilor nedeterminai
n studiul sistemelor de ecuaii difereniale de ctre
Jean Le Rond DAlembert(1717-1783);
1750- Gabriel Cramer d o regul de rezolvare a
sistemelor cunoscut sub denumirea de metoda lui
Cramer;
1755- sunt puse bazele calculului variaional de ctre
Lagrange(1736-1813) concomitent cu Euler,
1765- nceputul crerii geometriei descriptive de ctre
Gaspard Monge(1746-1818);
1766- crearea mecanicii analitice de ctre Joseph
Lagrange(1736-1813) cu enunarea principiului
vitezelor virtuale i a ecuaiilor Lagrange;
1767- demonstrarea iraionalitii lui de ctre
Heinrich Lambert(1728-1777);
1768- demonstrarea existena factorului integrant la
ecuaiile difereniale de ordinul nti de ctre
DAlembert;

143
1771- a fost dat ecuaia planului normal i formula
distanei dintre dou puncte din spaiu de ctre
matematicianul francez G. Monge;
1775- introducerea noiunilor de soluie general i
soluie particular n teoria ecuaiilor difereniale de
ctre Leonhard Euler; acesta a introdus i funcia
(n) - indicatorul lui Euler, precum i notaiile e, i,
f(x)i a creat teoria fraciilor continue;
1780- au fost introduse liniile de curbur ale
suprafeelor(G. Monge);
sunt descoperite funciile automorfe de
matematicianul francez Henri Poincare(1854-1912);
1785- a fost dat ecuaia planului tangent(G. Monge);
1796- este dat Teorema lui Fourier de determinare a
numrului rdcinilor reale cuprinse ntr-un interval, de
ctre Joseph Fourier(1768-183);
1797- este dat formula creterilor finite, cunoscut sub
denumirea de teorema lui Lagrange;
1798- au fost considerate cosinusurile directoare ale
unei drepte(G. Monge);
este introdus simbolul [.], pentru partea ntreag de
ctre Adrien Marie Legendre(1752-1833)
(1752-1833);
1807-, 1822 sunt date seriile Fourier care au contribuit
la crearea teoriei analitice a cldurii.
1812- este introdus seria hipergeometric de ctre Carl
Friedrich Gauss(1777-1855) matematician german, cel
care a demonstrat teorema fundamental a algebrei;
1816-1835- Augustin Cauchy(1789-1857), fondatorul
analizei matematice moderne, a enunat criteriul de
convergen al seriilor, criteriu care-i poart numele, a
dat primele teoreme de existen din teoria ecuaiilor
difereniale i al ecuaiilor cu derivate pariale, a

144
introdus noiunile de afix, modul al unui numr
complex, numere conjugate, transpoziie;
1820- introducerea noiunii de raport anarmonic de
ctre Chasles Michel(1793-1880), fondatorul
geometriei proiective alturi de matematicianul francez
Jean Poncelet;
1822
introducerea funciilor Bessel de ctre Friedrich
Bessel;
este introdus notaia pentru integrala definit
b

f ( x)dx , de ctre Fourier.;


a

este propus denumirea de reprezentare conform


de ctre Gauss;
cercul lui Euler sau cercul celor nou puncte este
considerat pentru prima dat de ctre Charles
Brianchon , Jean Poncelet i Karl Feuerbach,
atribuinduse din greeal numele lui Euler acestei
teoreme;
1823-1831- nceputul crerii primei geometrii
neeuclidiene de ctre Jano Bolyai(1802-1860)
concomitent i independent de cea a lui Lobacevski.
1824-
este dat denumirea de geometrie neeuclidian de
ctre Gauss;
Niels Abel(1802-1829) demonstreaz imposibilitatea
rezolvrii cu ajutorul radicalilor, a ecuaiilor algebrice
de grad mai mare dect patru;
1825- Abel introduce integralele ce-i poart numele;
1827- este creat teoria funciilor eliptice de ctre Abel;
1828
sunt introduse formele fundamentale ale suprafeelor
i curburii total a unei suprafee(curbura Gauss) de
ctre Gauss;
145
demonstrarea teoremei lui Fermat pentru n=5 de ctre
matematicianul german Dirichlet (1805-1859);
1830- este propus denumirea de grup cu nelesul
actual de ctre matematicianul francez Evariste
Galois(1811-1832);
1831- definitivarea calculului cu numere complexe de
ctre Gauss ;
1834- introducerea noiunii de factor de discontinuitate,
referitor la integralele
1837- introducerea notaiilor pentru limite laterale de
ctre Dirichlet i a funciei care i poart numele,
funcia Dirichlet;
W. Hamilton introduce termenul de asociativitate a
unei legi de compoziie;
1839- introducerea noiunii de integrale
multiple(Dirichlet);
1840- este dat o form a eliminantului a dou ecuaii
algebrice de ctre James Sylvester(1814-1897),
matematician englez;
1841- descoperirea invarianilor de ctre
matematicianul irlandez George Bole (1815-1864);
introducerea noiunilor de margine inferioar i
superioar ale unei funcii, de convergen uniform de
ctre Weierstrass(1815-1897);
1843- descoperirea cuaternionilor de ctre William
Hamilton (1805-1865);
1845- Teorema limit central este dat de
matematicianul rus Pafnuti Cebev;
1846- Legea numerelor mari Cebev;
introducerea variabilei complexe n teoria
numerelor imaginare de ctre DAlembert;
1847
este introdus calculul logic de George Boole,
creatorul algebrei booleene;

146
este introdus noiunea de ideal de ctre Ernest
Kummel(1810-1893);
1851- sunt introduse noiunile de rang i signatur a
unei forme ptratice i sunt propuse noiunile de
matrice i jacobian(J. Sylvester);
introducerea sufrafeelor riemann de ctre
matematicianul german Bernhard Riemann(1826-1866),
lui datorndu-se studiul integralei definite.
1852- introducerea segmentelor orientate AB de ctre
Chasles Michael(1793-188) care a formulat i
proprietile axei radicale a dou cercuri precum i a
conicelor i cuadricelor.
1853- Kronecker(1823-1891) introduce notaia
aij = det(aij ) ;
1854- este introdus noiunea de oscilaie ntr-un punct
de ctre Riemann care creeaz o nou geometrie
neeuclidian, numit geometria sferic;
1858- crearea calculului matriceal de ctre Arthur
Cayley(1821-1895) matematician englez ;
1871 Dedekind introduce noiunile de corp i modul
ceeace n limbajul actual exprim noiunile de subcorp
i Z-submodul ale lui C. Tot el introduce mulimea
ntregilor unui corp de numere algebrice, definind i
idealele acestei mulimi i demonstreaz teorema
fundamental de descompunere unic a oricrui ideal n
produs de ideale prime;
1872-
introducerea structurilor de subinel i modul de ctre
Dirichlet;
introducerea numerelor raionale prin teturi de
ctre Dedekind;

147
1873- Charles Hermite(1822-1901) demonstreaz
1
transcendena numrului e= lim(1 + ) n = 2,718281....
n n
1874- este dat denumirea de subgrup de ctre Sophus
Lie(1842-1899);
1874-1897- crearea teoriei mulimilor de ctre Georg
Cantor(1845-1918). El a introdus noiunile de mulime
deschis, mulime nchis, mulime dens, mulime bine
ordonat, mulime numrabil, punct de acumulare,
punct izolat, produs cartezian, reuniune, intersecie.
1878- rezolvarea problemei celor patru culori pentru
colorarea hrilor de ctre Cayley;
1880-sunt descoperite funciile automorfe de
matematicianul francez Henri Poincare(1854-1912);
1882- Ferdinand Lindemann(1852-1939) a
demonstrat trascendena numrului =3,141592......;
(un numr se numete transcedent dac nu este soluia
niciunei ecuaii algebrice cu coeficieni raionali); tot el
demonstreaz imposibilitatea cvadraturii cercului cu
rigla i compasul;
1893- H. Weber, asociaz conceptului de corp, sensul
de astzi, ca o structur cu o lege de grup aditiv i o
nmulire asociativ, distributiv i n care orice element
e inversabil;
1897- introducerea denumirii de inel de ctre
Hilbert(1862-1943);
1899 -axiomatizarea geometriei de ctre David
Hilbert;
1900- introducerea axiomatic a numerelor
ntregi(D.Hilbert);
1905- este introdus noiunea de distan ntre dou
mulimi nchise de ctre matematicianul romn Dimitrie
Pompeiu(1873-1954);

148
1910- este introdus denumirea de funcional de ctre
Jacques Hadamard (1865-1963), unul din fondatorii
analizei funcionale;
1912 -este descoperit noiunea de derivat
areolar(Pompeiu)
1927-s-a stabilit formula Onicescu referitoare la
geodezice dat de Octav Onicescu(1892-1983);
1928 -este introdus funcia areolar-conjugat de ctre
matematicianul romn Miron Nicolescu(1903-1975);
1933 -introducerea funciilor convexe de ordin superior
de ctre Tiberiu Popoviciu(1906-1975);
1936 -Matematicianul romn Gheorghe Mihoc(1906-
1981) d o metod cunoscut sub numele de metoda
Schulz-Mihoc, de determinare a legilor limit ale unui
lan Markov;
1941 -teorema lui Moisil referitoare la geodezicele unui
spaiu riemannian este introdus de Grigore
Moisil(1906-1973);
1944 -este introdus n domeniul algebrei moderne
noiunea de signatur de ctre matematicianul romn
Dan Barbilian(1895-1961);
1950 -este introdus noiunea de - derivat de ctre
Dan Barbilian;
1996 -celebra conjectur a lui Fermat este demonstrat
de ctre Andrew Wiles de la institutul Isaac Newton
din Cambridge.
2000 -este determinat cel mai mare numr prim 26972593-
1, avnd dou milioane de cifre, obinut cu ajutorul a 20
de mii de calculatoare puse n reea;

149
BIBLIOGRAFIE.
1: N. Mihileanu- Istoria matematicii,vol.1,vol2., Editura
tiinific i enciclopedic; Bucureti,1974/ 1981;
2. Vasile Bobancu- Caleidoscop matematic, Editura Niculesu;
3. Neculai Stanciu, 100 de probleme rezolvate. Editura Rafet;
4. Mic enciclopedie matematic, Editura Tehnic, Bucureti

150
Cuprins:

Breviar teoretic............................................................5
Probleme enunuri i rezolvri
Clasa a V-a.................................................................70
Clasa a VI-a................................................................81
Clasa a VII-a .............................................................92
Clasa a VIII-a ...........................................................116
Probleme propuse.................................................... 133
Istoricul noiunilor matematice............................... 141
152

S-ar putea să vă placă și